Pathology FinalA

March 22, 2018 | Author: vaegmundig | Category: Wound Healing, Healing, Ovarian Cancer, Inflammation, Hepatitis


Comments



Description

PATHOLOGY BOARD EXAMINATIONInstructions: Choose the best answer. 1. A 50-year old man has blood pressure of 150/90 mm Hg. If this person remains untreated for years, which of the following cellular alterations will be seen in the myocardium? A. atrophy B. metaplasia C. hypertrophy D. dysplasia Answer: C Reference: RPBD 7th p.7 2. On day 28 of her menstrual cycle, a 26-year old woman experiences menstrual bleeding which lasts for 5 days. She has had regular cycles for many years. Which of the following processes is most likely occurring in the endometrium just before onset of bleeding? A. apoptosis B. heterophagocytosis C. liquefactive necrosis D. hyperplasia Answer: A Reference: RPBD 7th p.26-27 3. At autopsy, a 60-year old man has an enlarged (2500 g) liver with yellow cut surface. Before death, his total serum cholesterol and triglyceride levels were normal, but he had decreased serum albumin concentration and prolonged prothrombin time. Which of the following activities most likely lead to these findings? A. playing golf B. cigarette smoking C. drinking gin D. aspirin ingestion Answer: C Reference: RPBD 7th p.35-36 4. A 70-year old woman suddenly lost consciousness and on awakening one hour later, she could not speak nor move her right arm and leg. Two months later, a head MRI showed a large cystic area in the left parietal lobe. Which of the following pathologic processes has most likely occurred in the brain? A. karyolysis B. fat necrosis C. apoptosis D. liquefactive necrosis Answer: D Reference: RPBD 7th p.22 6. A 21-year old woman complains of pain during menses (dysmenorrhea). To treat this symptom, she was advised to take indomethacin. This drug may help reduce her pain because it interferes with the production of which one of the following substances? A. bradykinin B. histamine C. leukotrienes D. estrogen Answer: A Reference: RPBD 7th p.65 7. What type of leukocyte actively participates in acute inflammatory processes and contains myeloperoxidase within its primary (azurophilic) granules and alkaline phosphates in its secondary (specific) granules? A. lymphocytes B. neutrophils C. eosinophils D. monocytes Answer: B Reference: RPBD 7th p.73 1 8. A 3-year old child touches a lit candle. Within several hours, there is marked erythema of the skin of the fingers on the child’s left hand, and small blisters appear on the finger pads. Which of the following terms best describe the process? A. fibrinous inflammation B. ulceration C. abscess formation D. serous inflammation Answer: D Reference: RPBD 7th p.76 9. A woman who is allergic to cats visits a friend who keeps several pet cats. During the visit, she inhales cat dander and within minutes, she develops nasal congestion and abundant nasal secretions. Which of the following substances is most likely to produce these findings? A. tumor necrosis factor B. bradykinin C. histamine D. complement C5a Answer: C Reference: RPBD 7th p.63 10. After receiving incompatible blood, a patient develops a transfusion reaction in the form of back pain, fever, shortness of breath and hematuria. Which of the following statements best classifies this type of immunologic reaction? A. systemic anaphylactic reaction B. complement-mediated cytotoxicity reaction C. systemic immune complex reaction D. delayed type hypersensitivity reaction Answer: B Reference: RPBD 7th p.210 12. A patient presents with a large wound on the left leg as a result of a vehicular accident. Initially his wound is filled with granulation tissue, which is composed of proliferating fibroblasts and new blood vessels (angiogenesis). Which of the following is a growth factor that is capable of inducing all the steps necessary for angiogenesis? A. vascular endothelial growth factor B. platelet-derived growth factor C. epidermal growth factor D. transforming growth factor Answer: A Reference: RPBD 7th p.95 13. A 16-year old patient has lacerations on his hand which required sutures. The sutures were removed after 10 days. Wound healing continued but the site was disfigured by a prominent, raised nodular scar which developed over the next 2 months. Which of the following terms best describes the process that occurred within this 2-month period? A. organization B. resolution C. keloid formation D. dehiscence Answer: C Reference: RPBD 7th p.114 14. A 17-year old woman receiving corticosteroid therapy for an autoimmune disease has an abscess on her upper outer left arm. She undergoes a drainage procedure but the wound heals poorly over the next month. Which of the following aspects of wound healing is most likely to be deficient in this patient? A. neutrophil infiltration B. collagen deposition C. re-epithelialization D. fibroblast elaboration Answer: B Reference: RPBD 7th p.114 15. Which of the following changes best describes the pathophysiology involved in the production of pulmonary edema in patients with congestive heart failure? A. widespread endothelial damage B. decreased plasma oncotic pressure C. acute lymphatic obstruction 2 D. increased hydrostatic pressure Answer: D Reference: RPBD 7th p.122 16. A 23-year old medical student develops a “red” face after being asked a question during a lecture. Which of the following statements best describes the vascular reaction? A. active hyperemia B. petechial hemorrhage C. allergic reaction D. passive congestion Answer: A Reference: RPBD 7th p.122 17. While preparing her lunch, a housewife nicks her finger with a knife. Seconds after the injury, the bleeding stops. Which of the following mechanisms is most likely to reduce blood loss from a small dermal arteriole? A. protein C activation B. vasoconstriction C. neutrophil chemotaxis D. platelet aggregation Answer: B Reference: RPBD 7th p.124 18. A 26-year old rock climber fractures his left femur after falling from a height of 20 meters. He was hospitalized and over the next several days develops progressive respiratory problems. He died three days later. At the time of autopsy, oil red-O positive material is seen in the small blood vessels of the lungs and brain. Which of the following is the most likely diagnosis? A. air emboli B. padoxical emboli C. fat emboli D. saddle emboli Answer: C Reference: RPBD 7th p.136 19. A 23 –year old who had a myocardial infarction 1 year a year ago now has chest pain when exercising. His underlying disease is due to an absence of LDL receptors on liver cells, inherited as an autosomal dominant condition. Which of the following laboratory findings is most likely to be present in this patient? A. ketonuria B. hypercholesterolemia C. hypoglycemia D. abetalipoprotenemia Answer: B Reference: RPBD 7th p.156 20. A 20-year old woman of average intelligence and short stature is being evaluated for primary amenorrhea. Physical examination reveals a shield-shaped chest and her elbows turn outward when her arms are at her sides. She has a “thick neck” and you notice the absence of secondary female sex characteristics. Serum estrogen levels are found to be decreased, while FSH and LH levels are increased. Which of the following is the most likely diagnosis? A. Turner’s syndrome B. fragile X syndrome C. Klinefelter’s syndrome D. XYY syndrome Answer: A Reference: RPBD 7th p.179 23. A 26-year old woman presents with increasing fatigue, arthritis, shortness of breath, and a bimalar, photosensitive erythematous rash. Biopsies from this rash reveal liquefactive degeneration of the basal layer of the epidermis with a perivascular lymphoid infiltrate. Physical examination shows bilateral pleural effusions, the fluid from which when examined histologically reveals multiple erythrocytes phogocytized by phagocytic leukocytes. Which of the following is the most likely diagnosis? A. dermatomyositis B. systemic lupus erythematosus C. Sjogren’s syndrome 3 This was diagnosed as an inguinal hernia. rheumatoid arthritis Answer: B Reference: RPBD 7th p. papilloma Answer: B Reference: RPBD 7th p. No other family members are affected by this disorder. Cytologically. adipose tissue was sent to the laboratory for examination. there is a 2-cm firm nontender nodule on the left lobe of the thyroid gland. Vitamin K Answer: D Reference: RPBD 7th p. A fine needle biopsy of the nodule reveals cells consistent with thyroid carcinoma. Which of the following is the basic defect causing this change in the liver? A. A 36-year old man presents with a new onset of a “bulge” in his right inguinal area. Vitamin C D.319 27. A 33-year old woman undergoes an annual physical examination and there are no abnormal findings. exposure to polyvinyl compounds B. The resected hernial sac contained some adipose . Which of the following statements best explains the significance of these findings? A. uniform population of cells Answer: B Reference: RPBD 7th p. the cells obtained from the cervical smear show severe epithelial dysplasia. a “moon face. and an enlarged fatty liver. Which of the following features should be present to make a diagnosis of lipoma rather than normal adipose tissue? A. On physical examination. An ovarian teratoma is present C.270 26.275 29. Which of the following is most relevant in the woman’s past medical history? A.227 25. A 64-year old man has several episodes of hematuria in the past week. Which of the following terms best describes this lesion? A. Vitamin B C. A urinalysis shows 4+ hematuria and cytologic examination of the urine shows that atypical cells are present. there are no abnormal findings. anaplasia B. Antibiotic therapy will cure the lesion Answer: A Reference: RPBD 7th p. On physical examination. A urologist performs a cystoscopy and observes a 4 cm mass with a nodular.D.271 28. Decreased carbohydrate intake leads to hypoglycemia C. carcinoma C. The use of broad-spectrum antibiotics can produce a bleeding diathesis characterized by hematomas. prominent nucleoli D. history of trauma Answer: C Reference: RPBD 7th p. fibrous capsule C. melena and bleeding gums by decreasing the normal gut flora and inducing a deficiency of which one of the following vitamins? A. A Pap smear was done as part of the pelvic examination. Vitamin A B. fibroma D. Decreased caloric intake leads to hypoalbuminemia D. radiation exposure in childhood D. Decreased protein intake leads to decreased lipoproteins Answer: D 4 . The lesion could progress to invasive cervical carcinoma B.456 33. sarcoma B. ulcerated surface in the dome of the bladder. A 23-year old woman has noted a palpable nodule on her neck for the past 3 months. There has been a regression of cervical carcinoma D. An apathetic male infant from the slums is found to have peripheral edema. Decreased fluid intake leads to hypernatremia B. chronic alcoholism C. Histologic sections reveal caseous necrosis surrounded by acid fast bacilli and few scattered giant cells. fever. Within 2 days. atypical lymphocytes C. There are no external malformations. A 38-year old man presents with cough. erythroblastosis fetalis Answer: B Reference: RPBD 7th p. paralysis C. hyaline arteriolosclerosis C. giant cell arteritis D.521 45. A chest x-ray reveals irregular densities in the upper lobe of the right lung. A few days later. Over the next 2 weeks. A 40-year old woman presents with recurrent severe headaches and increasing visual problems. Which of the following conditions occurring during gestation would most likely produce these findings? A. cryptosporidiosis Answer: C Reference: RPBD 7th p. congenital cytomegalovirus D. epithelial cells Answer: A Reference: RPBD 7th p.399 40. Her symptoms are most likely to be associated with which one of the following abnormalities? A.447 38.368 39. Physical examination reveals her blood pressure to be 220/150. pregnancy induced hypertension C. blastomycosis C. low density lipoproteins B. congestive heart failure D. medial calcific sclerosis B. Which of the following clinical manifestations of this infection is most likely to appear decades later? A. night sweats and weight loss.5 cm reddish macule. Throughout infancy.381 41. the infant’s head size is normal. Laboratory evaluation of his serum also discloses hypertriglyceridemia. intermediate density lipoproteins Answer: B Reference: RPBD 7th p. A 21-year old woman gives birth at term to an infant weighing 1859 g. The mucicarmine stain reveals numerous yeasts that stain bright red. but the crown-heel length and foot length are reduced. the vesicles ruptured and crusted over. developmental milestones are delayed. the macules became vesicle. cryptococcosis D. A 36-year old man with AIDS presents with deteriorating mental status. After appropriate work-up. A 5-year old girl developed a rash over her chest that began as 0. chylomicrons D. A spinal tap was done. These giant cell are a result of fusion together of what type of cell? A. maternal diabetes mellitus B. which is due to his diabetes. he is diagnosed as having type II diabetes mellitus. infertility Answer: A Reference: RPBD 7th p. crops of the lesion spread to the face and extremities. On physical examination. which is due in part to insulin resistance. shingles B. chromomycosis B. activated macrophages B. Which of the following is the most likely diagnosis? A.477 44. The cerebrospinal fluid was stained with mucicarmine and India ink.Reference: RPBD 7th p. A 40-year old obese man presents signs and symptoms of hyperglycemia. Takayasu’s arteritis Answer: C 5 . The India ink demonstrates the capsule of the organism. The most common type of secondary hyperlipidemia associated with diabetes mellitus is characterized by elevated serum levels of which of the following substances? A. very low density lipoproteins C. reactive fibroblasts D. contraction atelectasis D. then moving downward into the abdomen. He describes the pain as beginning in the anterior chest. and painless enlargement of several supraclavicular lymph nodes. Which of the following cellular components is most likely to be present in this mass? A. A 21-yearold woman presents with fever. A 6-year old girl accidentally inhales a plastic bead. Myocardial infarction is suspected but was ruled out due to normal ECG. yolk sac cells B. non-Hodgkin’s lymphoma Answer: A Reference: RPBD 7th p. patchy atelectasis Answer: A Reference: RPBD 7th p. the left testis is three times the size of the right testis and is firm on palpation. congestive heart failure D.Reference: RPBD 7th p. An x-ray of the patient’s abdomen shows a “double-barrel” aorta. His blood pressure is found to be 160/115. and a widely split S2. Laboratory examination finds decreased serum ferritin. viral pneumonia C. radiating to the back. calf tenderness. compression atelectasis C. On physical examination. which lodges in one of her bronchi. Which of the following pulmonary abnormalities is most likely present in this girl? A. pulmonary embolus Answer: D Reference: RPBD 7th p. Leydig cells C. the levels of which are inversely proportional to the serum levels of which one of the following substances? A. A chest x-ray reveals the mediastinum to be shifted toward the side of the obstruction. Which of the following is the most likely diagnosis? A. cytotrophoblasts Answer: A 6 . Laboratory studies include an elevated alpha fetoprotein level. A biopsy from one of the enlarged lymph nodes shows binucleate giant cell with prominent acidophilic “owl-eye” nucleoli. reactive hyperplasia C. Which of the following is the cause of the abnormality? A. Physical examination finds the patient to be afebrile with moderate respiratory distress. A 62-year old woman presents with increasing fatigue and is found to have hypochromic normocytic red cells in her peripheral smear. While recovering in bed 1 week after an abdominal hysterectomy. seminoma cells D. 713 56. atelectasis B. hemosiderin D. bilirubin C. resorption atelectasis B.742 64. cat-scratch disease D. Hodgkin’s disease B. weight loss. Physical examination finds her heart rate and respiratory rate to be increased in frequency. A 35-year old man noticed an increased feeling of heaviness in his scrotum for the past 10 months. night sweats. transferring Answer: D Reference: RPBD 7th p. a 43-year old woman develops acute shortness of breath with hemoptysis. Also present are atypical mononuclear cells that are surrounded by clear spaces (lacunar cells).643 52.536 46. A 61-year old man presents with the sudden onset of excruciating pain. abnormal collagen synthesis D. Which of the following is the most likely diagnosis? A. haptoglobulin B.531 50. 686 54. congenital defect in the aorta wall C. loss of elastic tissue in the media B. a microbial infection Answer: A Reference: RPBD 7th p. Over the next year. A 47-year old man presents with increasing “heartburn” especially after eating or when lying down. normal appearing appendix B. Reflux esophagitis D. squamous cell carcinoma Answer: B Reference: RPBD 7th p.924 70. A liver biopsy reveals chronic inflammation in the portal triads that focally destroys the limiting plate and “spills over” into the adjacent hepatocytes. Which of the following is the most likely histologic diagnosis for this colonic mass? A. Which of the following is the expected microscopic appearance of this appendix? A.1043 65. adenocarcinoma C. obstructive jaundice and a dilated gallbladder C. Physical examination is unremarkable and fecal occult blood is negative. leiomyosarcoma B. A 60-year old man presents with increasing fatigue. systemic lupus erythematosus D. a pancreatic cancer located in the head of the pancreas should be suspected in an individual with which one of the following clinical signs? A. Biopsies taken approximately 4 cm proximal to the gastroesophageal junction reveal metaplastic columnar epithelium. Which of the following is the most likely diagnosis? A. mild fever and an elevation of peripheral leukocyte count. migratory thrombophlebitis B. obstructive jaundice and a nonpalpable gallbladder Answer: B Reference: RPBD 7th p. yellow tumor at the tip of the appendix D. her liver enzymes have remained elevated and all serological tests for viral markers remain negative. angiosarcoma B. hamartomatous polyp B. neutrophils within the muscular wall Answer: D Reference: RPBD 7th p. Work-up reveals elevated levels of alpha fetoprotein in this patient’s blood. An appendectomy is performed. An LE test is also positive.870 67. Anti smooth muscle and antinuclear antibodies are present. Which of the following is the most likely diagnosis? A. His past medical history is unremarkable. Physical examination is also unremarkable except for positive occult blood in stool and microcytic hypochromic anemia. cholangiocarcinoma D.804 66. nausea and vomiting. CT scan of the abdomen reveals a single mass in the left lobe of the liver.Reference: RPBD 7th p. A 48-year old woman with increasing fatigue and is found to have elevated liver enzymes (AST and ALT). Barium enema study shows a 4 cm mass in the left side of his colon having an “apple core” appearance. autoimmune hepatitis B. hepatocellular carcinoma Answer: D Reference: RPBD 7th p. A 57-year old male alcoholic with a history of chronic liver disease presents with increasing weight loss and ascites. dilated lumen filled with mucus C. Which of the following is the most likely diagnosis? A. chronic persistent hepatitis Answer: A Reference: RPBD p. Mallory Weiss syndrome Answer: B Reference: RPBD 7th p.862 68. Barrett’s esophagus C. primary biliary cirrhosis C. hepatoblastoma C. carcinoid tumor D. steatorrhea and a tender gallbladder D.903 69. A 20-year old woman with abdominal pain localized to the right lower quadrant. According to Courvoisier’s law.948 7 . The lesion is completely contained within the duct and no invasion into underlying tissue is seen.72. these cysts are lined by tall columnar epithelium. Which of the following is the correct diagnosis for this ovarian tumor? A. ductal papilloma Answer: D Reference: RPBD 7th p. A 30-year old woman in the third trimester of her first pregnancy develops persistent headaches and swelling of her legs and face. acromegaly B. clear cell tumor Answer: A Reference: RPBD 7th p. diabetes C. metastatic ovarian carcinoma C. The respiratory alkalosis in this individual caused tetany by decreasing the ionized serum levels of what substance? A. Sections from the mass reveal multiple fibrovascular cores lined by several layers of epithelial cells. magnesium D. Physical examination reveals a 1 cm freely moveable mass that is located directly beneath the nipple. and he also thinks that his hand and jaw are now slightly larger. and large hands and feet. the mass consists of multiple cystic spaces.1106 77. A 22-year old woman presents with pelvic pain and is found to have an ovarian mass that measures 3 cm in diameter. A previous physician told her that she may have “chocolate in her cysts”.1188 8 . gigantism D. subserosal leiomyoma Answer: C Reference: RPBD 7th p. decreased bicarbonate. Physical examination finds an anxious woman with increased respiratory rate . endometrioid tumor D. with some of the cells being ciliated. Initial laboratory tests reveals increased serum glucose. Which of the following is the most likely diagnosis? A. endometriosis D. preeclampsia D. Paget’s disease C. Atypia is minimal. mucinous tumor C. Histologically. nephrotic syndrome B. Which of the following will most likely be present in this patient? A. Physical examination reveals a prominent forehead and lower jaw. A 24-year old woman for a fertility work-up gives a history of severe pain during menses.1126 79. and an increased blood pH. Which of the following is the most likely diagnosis? A. A 20-year old woman presents with the acute onset of perioral tingling and muscle cramps involving both of her hands. serous tumor B. calcium B. ectopic pregnancy Answer: C Reference: RPBD 7th p. benign phyllodes tumor B. however. He states that recently he has had to change his shoe size from 8 to 9. now her blood pressure is 170/105 mmHg and urinalysis reveals slight proteinuria. sodium C.1095 74.1161 82. intraductal carcinoma D. enlarged tongue. apoplexy Answer: A Reference: RPBD 7th p. while laboratory examination reveals decreased arterial pCO2. A 43-year old man presents with increasing fatigue and occasional headaches. potassium Answer: A Reference: RPBD 7th p. gestational trophoblastic disease C. A 41-year old woman presents with the new onset of bloody discharge from her left nipple. acute pelvic inflammatory disease B.1083 73. Grossly. Which of the following is the most likely explanation for this group of findings? A. Early during her pregnancy her prenatal consultations were unremarkable. Additionally. He was then brought to the hospital and was assessed as having an epidural hematoma. polymyositis D. and histologic sections reveal infiltrating groups of cells in the dermis.1359 9 . Rhabdomyolysis B. The patient does not complain of difficulty in chewing food or talking. neck and trunk. He is not taken to the hospital. mobile.1336 87. myotonic dystrophy B. A 62-year old woman notices a lump on he left side of her face that has become larger over the past year. A 22-year old male gets hit on the head by a hard object in the temporal area. The oral mucosa appears normal. Laboratory evaluation finds increased plasma levels of cortisol and glucose. Which of the following is the most likely diagnosis? A. ulcerated lesion located on the pinna of his left ear. his calves appear to be somewhat larger than normal.1344 89. Lack of lactate production during ischemic exercise C. sialolithiasis D. These cells have eosinophilic cytoplasm. Which of the following is the most likely diagnosis? A. dermatofibrosarcoma protuberans B. Physical examination reveals that the boy uses his arms and shoulders to rise from the floor or a chair. squamous cell carcinoma Answer: D Reference: RPBD 7th p. Transection of a branch of the middle meningeal artery B. a 4 to 5 cm firm. A 61-year old woman presents with difficulty swallowing. Which of the following conditions is most likely to account or these findings? A. and diplopia. Merkel cell carcinoma C. Which of the following is the most likely cause of this woman’s disease: A. A 74-year old man with a slowly growing. painless mass is palpable in he region of the left parotid gland. By evening he develops severe headache with vomiting and confusion. ptosis. Addison’s disease B. Conn’s syndrome Answer: C Reference: RPBD 7th p. Cushing’s syndrome D.1207 86. The excess adipose tissue is mainly distributed in her face. mucoepidermoid carcinoma Answer: A Reference: RPBD 7th p. pleomorphic adenoma B. On physical examination. and intracellular keratin formation. The lesion is excised. Corticosteroid therapy Answer: C Reference: RPBD 7th p. Duchenne’s muscular dystrophy C. Antibodies to the acetylcholine receptor D. Cortical bleeding opposite point of traumatic injury Answer: A Reference: RPBD 7th p. Which of the following is the most likely diagnosis? A. a waddling gait.1241 92. Bleeding from torn bridging veins C. He does not lose consciousness but afterwards develops a slight headache. A 56-year old woman presents with increasing muscle weakness and fatigue. and difficulty climbing steps. Rupture of a preexisting berry aneurysm D. Sjogren syndrome C. Which of the following is most likely present in this individual? A.777 95. Bartterr’s disease C.83. Physical examination finds an obese adult woman with purple abdominal stria and increase facial hair. A 6-year old boy presents with clumsiness. inclusion body myositis Answer: B Reference: RPBD 7th p. intercellular bridges. basal cell carcinoma D. is best characterized by which one of the following clinical signs? A. Huntington’s disease D. extension and internal rotation of upper limb D. What is the diagnosis? (Robbin’s 5th editionp 461) a.1331 98. cobalamin D. Physical examination examination reveals absent deep tendon reflexes. Guillain-Barre syndrome B. Toxoplasma gondii Answer: B Reference: RPBD 7th p. There is also paralysis of the lateral rectus muscle. A 22-year old man has a mild pharyngitis followed a few days later by sudden onset of a severe headache. A 51-year old man with acute psychosis has a long history of alcoholism. Metachromatic leukodystrophy Answer: B Reference: RPBD 7th p. syringomyelia C. but very few cells are present. paraganglioma c. and a lumbar puncture the CSF protein to be increased. Physical examination reveals atrophy of the muscles of both hands.96.1396 97. Pain in thumb. Weakness of extensors of wrist and fingers Answer: B Reference: RPBD 7th p. middle finger and lateral half of ring finger C. Which of the following is the most likely diagnosis? A. produced by damage to or pressure on the median nerve deep to the flexor retinaculum. riboflavin Answer: A Reference: RPBD 7th p. A 47-year old man presents with weakness and cramping that involves both hands. Physical examination shows nuchal rigidity. Which of the following is the most likely diagnosis? A. thiamine B. hyperactive reflexes and muscle fasciculations involving the arms and legs and positive Babinski reflex. Poliovirus D. Brown-Sequard syndrome B.1369 99. A biopsy of a peripheral nerve reveals inflammation and demyelination. His temperature is 38. diabetes melitus Answer: C Reference: RPBD 7th p. melanoma d. retinoblastoma b. Which of the following is most likely to be the etiologic agent? A. After recovering from a viral respiratory infection.423 100. 1335 Reference: Robbins Pathologic Basis of Disease 7th Edition 1.8C. Carpal tunnel syndrome. Numbness in the fifth finger and medial portion of the ring finger B. Adduction. Sensation appears to be normal in both arms and legs. immature teratoma 10 . The gross appearance of his brain shows extensive purulent exudate covering the brain convexities. a 22-year old woman presents with weakness in her distal extremities that rapidly ascends to involve proximal muscles. niacin C. Gullain-Barre syndrome D. A 5-year old male child presented with strabismus. A deficiency of which of these nutrients is most likely the cause of these findings? A. Amyotrophic lateral sclerosis C. Neisseria meningitidis C. Cryptococcus neoformans B. He keeps on talking nonsense and is unable to follow simple commands. poor vision and whitish hue to the pupil of the left eye. Enucleation was done and biopsy revealed FlexnerWintersteiner rosettes. index finger. 24-72 hrs d. concentric calcific concretion d. stage IV 8. Biopsy was done which revealed inflammatory cells within the lamina propria and beginning atrophy of the glandular structures. celiac sprue d. 18-24 hrs b. stage II c. tall. CXR showed apical lesion in the right lung. central caseous necrosis b. A 70-year old male died of myocardial infarction. EGD showed red gastric mucosa. 4-12 hrs c. langhans type giant cells c. What is the stage of the disease? (Robbins 5 th ed p 643) a. stage I b. none of the above 3. stage III d. A 4-year old male with non-hodgkin’s lymphoma presented with paraaotic lymphadenopathy and splenic mass. stage I b. none of the above 6. The macrophages contain PAS (+) granules and rod-shaped bacilli by electron microscopy. stage III d. What is the stage of the disease? ( Robbins 5 th ed p 1107) a. immunologic c. Autopsy was done and microscopic findings of the myocardium revealed coagulative necrosis with loss of nuclei and striations of the cells. The following are major etiologic associations of this disease except: (Robbins 5th ed. whipple’s disease c. tropical sprue b. A 35-year old male had been having on and off upper abdominal discomfort and vomiting for 1 year. What is the diagnosis? ( Robbins 5th ed p 726) a. columnar to cuboidal epithelial cells that line up along alveolar septa and project into the alveolar spaces in numerous branching papillary projections. How old is the lesion? (Robbins 5 th ed p 536) a. Punch biopsy of the small intestine showed mucosa laden with distended macrophages in the lamina propria. disaccharidase deficiency 7. stage IV 11 . A 50-year old female presented with diarrhea and weight loss for 5 years. Histologic sections of the peripheral lung mass of a 60-year old female showed distinctive. What is the diagnosis: (Robbins 5 th ed p 799) a. small cell carcinoma b.2. bronchioalveolar carcinoma d. 3-7 days 4. stage II c. The following are histologic features of this disease entity EXCEPT: (Robbin’s 5 th edition p 81) a. p 771) a. chronic infection d. A 54-year old male was brought to the emergency room due to massive hemoptysis. A 40-year old female presented with a 4 cm fixed right breast mass associated with movable axillary nodes. alcohol consumption and cigarette smoking b. squamous cell carcinoma c. large cell carcinoma 5. ( Robbins 5th ed p. 572-576). Page 50 12 . Page 13 4. Biopsy of the lung most likely reveal which of the following: A. multiple myeloma of bone B. Robbins 7th ed. Endometrial carcinoma d. Page 22 5. Robbins 7th ed. Robbins 7th ed. fatty change C. Pancreatic adenoma 10. a. C5a C. Ovarian carcinoma c. Hodgkin’s lymphoma CELL INJURY AND ADAPTATION 1. absence of inflammation B. A. Which of the following substances may cause edema during inflammation? A. Page 39 3. A 45 y/o female has a history of regular oral contraceptive intake. Which is characteristic of irreversible injury? A. A 27 y/o male with fever and leokocytosis expired and was sent for autopsy. cell membrane defects Ref. histamine B. CXR shows an ill-defined mass along the apex of both lobes. surface blebs D. Paget disease Ref. Hepatic adenoma b. interleukine-1 D. cell swelling B. Which of the following is a feature of necrosis? A. intact plasma membrane D. TNF Ref. One of the gross finding included multiple pale. leaking of enzymes Ref. These are eosinophilic cytoplasmic inclusions seen in alcoholic liver disease. parathyroid adenoma D. Robbins 7th ed. Kimura bodies Ref. papillary thyroid carcinoma C. She has a higher risk of developing this tumor compared to non-users. Robbins 7th ed. cell swelling D. The most likely diagnosis is:(p. Metastatic carcinoma b. caseation necrosis D. Psammoma bodies are foci of dystrophic calcification seen in which condition? A. Page 12 ACUTE AND CHRONIC INFLAMMATION 7. Disseminated histoplasmosis c. affects numerous cells at a time C. liquifactive necrosis B. reduced cell size B. Page 26 6. Russel bodies D. intact cellular contents C. Page 41 2. Sputum AFB was positive. karyolysis Ref. Robbins 7th ed.9. Robbins 7th ed. wedge-shaped infarcts all over the spleen.416) a. A 40 year old male complains of chronic cough and weight loss. Amyloid B. Infective endocarditis d. Mallory bodies C. Which of the following is characteristic of apoptosis? A. coagulative necrosis C. tumor necrosis Ref. fibronectin Ref. eosinophils Ref. compound fracture fixation B. lymphocytes C. This is the most abundant glycoprotein in basement membrane. granulomatous C. Which of the following is an example of wound healing by primary intention? A. basophils C. Type of inflammation induced by indigestible foreign bodies. Robbins 7th ed. A. Robbins 7th ed. C6-7 C. C3b B. endoscopic surgery D. neutrophils B. contraction D. Robbins 7th ed. Page 79 12. 6 months Ref. The hallmark of acute inflammation is: transient vasoconstriction B. Robbins 7th ed. This is known as the membrane attack complex. Robbins 7th ed. neutrophils D. Page 56 10. fibrinous Ref. abscess drainage Ref. granuloma Ref. keloid C. A. elastin B. tissue remodeling C. vasodilation C. Page 114 17. contracture D. A. platelet derived growth factor C. A wound will attain maximum strength at what time? A. 3 months D.8. Robbins 7th ed. A. serous B. Which of the following white blood cells predominate within 6 to 24 hrs after tissue injury? A. laminin C. Central actors in chronic inflammation. Robbins 7th ed. fistula repair C. Page 113 16. Page 111 13 . re-epithelialization Ref. ulcer B. lymphocytes Ref. suppurative D. Inadequate formation of granulation tissue or scar can lead to which of the following? A. macrophage B. Robbins 7th ed. Epidermal growth factor B. Reduction in size of large wounds is attributed to which of the following? A. fibrosis B. Robbins 7th ed. slowing of circulation Ref. 1 month C. A. Page 114 15. Page 83 11. This is a potent angiogenic growth factor. C1b-3 Ref. Robbins 7 ed. macrophages D. Page 105 18. proteoglycan D. C5b-9 D. fibroblast growth factor D. 2 weeks B. increased vascular permeability D. transforming growth factor th Ref. Robbins 7th ed. Page 64 9. Page 111-112 14. Page 50 WOUND HEALING AND TISSUE REPAIR 13. underlying conditions is he most likely to have. macrophages Ref. A 70 year old male after spending a month in the hospital confined to a wheel chair due to arthritis . dsDNA C. and later went into shock. CXR showed bilateral pleural effusion while abdominal ultrasound showed ascites with small liver.? A. heart failure. A 40 year old chronic alcoholic complains of easy fatigability and orthopnea at night. & hypercoagulability Ref. Robbins 7th ed. sepsis Ref. anaphylactic C. Virchow’s triad (factors favoring thrombosis) includes w/c of the ff. Page 139 IMMUNE SYSTEM 25. occult cancer. IgM D. Page 201 14 . if the gross appearance of the liver shows "nutmeg" pattern? A. lymphatic obstruction D. & jaundice C. He later lost consciousness and his BP rapidly declined. Robbins 7th ed. A. A 32 year old male experienced difficulty of breathing and edema of the lungs after being injected with a new antibiotic drug. thrombocytosis. increased hydrostatic pressure B. Systemic Lupus Erythematosus can be confirmed by testing for which antibodies? A. idiopathic D. Robbins 7th ed. hyperglycemia. Robbins 7th ed. Robbins 7th ed. pulmonary embolism B. Robbins 7th ed. vascular injury. & erythrocytosis B. liver C. W/c of the ff. This antibody mediates hypersensitivity reactions and is most commonly found on surface of mast cells. A. lungs D. old age C. & hypertension D. Page 206 26. He also has increasing lower leg swelling w/ grade 2+ pitting edema to the knees. Page 121 20. chronic hepatitis C. Page 136 21. bronchopneumonia Ref.. T cells C. bone Ref. sepsis D. SS-A B. bile duct obstruction Ref. Page 122 23. kidney B. suddenly experiences difficulty of breathing after walking on his own. portal vein thrombosis B. stasis. This is due to which of the following? A. anti-GBM Ref. hemorrhage B. B cells B. hyperlipidemia. An arterial thromboemboli is most likely to produce a hemorrhagic infarct in which organ? A. Robbins 7th ed. The serum AST & ALT are observed to be increasing in a 61 y/o man over the past wk.FLUID AND HEMODYNAMIC DISORDERS 19. Page 130 24. Robbins 7th ed. IgE Ref. Page 228 27. What is the mechanism of effusion? A. IgG C. congestive heart failure D. He has prominent jugular venous distention in neck veins to the level of the mandible. These cells can lyse tumor cells or virus infected cells without prior sensitization. inflammation of membranes Ref. What is the most likely diagnosis? A. NK cells D. leukocytosis. Page 138 22. IgA B. decreased plasma oncotic pressure C. centromere D. Robbins 7th ed. albinism B. Page 324 34. Cushing syndrome C. tuberous sclerosis B. galactosemia th Ref. type III D. iodine D. iron C. A. Page 339 B. type II C. nickel Ref. 6 and 11 Ref. Robbins 7th ed. type I B. A 10 year old female consulted due to multiple neurofibromas. asbestos D. 20 and 40 D. Human papilloma virus type responsible for cervical carcinoma. Page 272 36. well-differentiated B. Page 212 GENETIC DISORDERS 29. adenocarcinoma Ref. A. Robbins 7th ed. Robbins 7th ed. Page 461 15 . A patient with myasthenia gravis developed progressive muscle weakness. A. copper th Ref. Robbins 7th ed. A. This is what type of hypersensitivity reaction? A. Robbins 7th ed.28. Robbins 7 ed. Robbins 7th ed. Robbins 7 ed. cutaneous café au lait macules and pigmented iris hamartomas. A. 16 and 18 C. Page 334 35. ochronosis D. slow growing C. chromosome 5pD. vinyl chloride C. seminoma C. Page 279 37. hypercalcemia B. A. This condition is called A. Page 285 33. Page 175 31. Deficiency in homogentisic oxidase results in this condition which manifests with black discoloration of the urine. carcinoid syndrome Ref. A. Most common paraneoplastic syndrome associated with squamous cell carcinoma of the lung. type IV Ref. metastasis Ref. SIADH D. Robbins 7th ed. CA-125 C. phenylketonuria C. embryonal carcinoma D. zinc B. A. Extremely radio-sensitive testicular tumor that tends to spread to lymph nodes along the iliac arteries and aorta. Tumor marker for breast cancer. trisomy 18 B. Marfan’s syndrome D. Robbins 7th ed. A. arsenic B. Von Recklinghausen’s disease C. Which of the following is characteristic of a malignant tumor. CA-20-1 ENVIRONMENTAL 38. Turner syndrome Ref. This is the most common genetic cause of mental retardation. Robbins 7th ed. cohesive and well-demarcated D. CA-15-3 Ref. Chemical carcinogen implicated in the causation of mesothelioma. Page 167 30. Pages 168-169 NEOPLASIA 32. Von Hippel-Lindau syndrome Ref. This nutrient deficiency presents with microcytic and hypochromic anemia. trisomy 21 C. teratoma B. CA-19-9 D. 2 and 4 B. neuroblastoma D. A. Robbins 7th ed. caffeine Ref. Robbins 7th ed. Mycobacterium tuberculosis B. Robbins 7th ed. head injury B. Page 394 16 . Pages 487-488 44. Vitamin D D. miliary C. Giemsa D. Page 424 DISEASE OF INFANCY AND CHILDHOOD 42. Vitamin deficiency results in night blindness and xeropthalmia. melanoma Ref. amniotic fluid aspiration Ref. Robbins 7th ed. Vitamin B1 Ref. ducreyi Ref. gram stain B. chancre B. phenylketonuria B. A. measles virus B. A. leukemia C. hyaline membrane disease C. trachomatis C. A. Page 389 49. ethanol D. Page 450 40. silver stain C. A. incision Ref. primary focus D. Page 362 47. laceration D. Injury caused by a blunt force that damages small blood vessels. Clostridium perfringens D. Page 499 INFECTIOUS DISEASES 45. Robbins 7th ed. abrasion B. acid-fast stain Ref. A. Robbins 7th ed. This is the most frequent childhood tumor causing death in children younger than 15 years old. contusion C. gonorrhea B. Chronic ingestion of this substance by mothers may cause growth retardation and microcephaly in infants. Page 385-386 48. astrocytoma B. Page 481 43. T. Special stain used in diagnosing mycobacteria and nocardiae infections. In-born error of metabolism associated with severe mental retardation and strong mousy odor to the affected infant. Which of the following organisms cause necrotizing inflammation.39. Robbins 7th ed. Robbins 7th ed. Vitamin C C. Vitamin A B. pallidum D. and are most commonly seen in the bone. warts Ref. gumma D. Most common cause of respiratory distress in the newborn. cavitating B. Page 361 46. scrofula C. usually without disruption of the continuity of tissues. A. Robbins 7th ed. skin and mucous membranes of the mouth. A. galactosemia C. cord coiling D. Robbins 7th ed. Robbins 7th ed. C. lactose C. N. A. Pott’s disease Ref. Staphylococcus aureus Ref. iron B. cystic fibrosis D. tyrosinemia Ref. H. A. A. This obligate intracellular gram negative bacterium is the most common bacterially sexually transmitted disease in the world. Page 443 41. Pulmonary disease caused by mycobacteria characterized by small visible ( 2mm) foci of yellow white consolidation scattered through the lung parenchyma. These nodular lesions are related to delayed hypersensitivity to Treponema pallidum. sarcoidosis B. left ventricle Ref. Robbins 7 ed. A. Lymphocyte predominant D. arrhythmia D. papilloma Ref. Which of the following is characteristic of emphysema? A. thalassemia B. Robbins 7th ed. A. leg C. mucous plugs containing eosinophils C. A. Pages 717-718 59. Robbins 7 ed. A. myxoma C. This type of pneumonia is characterized by patchy consolidation of the lung with foci of acute suppurative inflammation that are poorly defined grossly. Lymphocyte depleted Ref. Most common cause of death after myocardial infarction. Page 613 HEMATOPOIETIC AND LYMPHOID DISEASES 53. myocarditis C. bronchopneumonia D. A 20 year old male with rheumatic heart disease suddenly collapsed after a basketball game. right atrium B. Robbins 7th ed. This is the most common primary cardiac tumor in adults A. Robbins 7th ed. hypotension C. Page 737 55. congestive heart failure B. Most common type of Hodgkin’s Lymphoma characterized by broad collagen bands separating lymphoid tissue into well-defined nodules. angiosarcoma D. R-S cells C. Noncaseating granulomatous inflammation in the lymph nodes and skin is typical in what condition? A. Robbins 7th ed. chronic lymphadenitis Ref. Robbins 7 ed. helper T-cells B. tuberculous Ref. Mixed cellularity C. tuberculosis C. sickle cell anemia D. histoplasmosis D. lipoid pneumonia C. aorta D. Robbins 7th ed. myocardial rupture th Ref. calcified aorta D. Which of the following is a common feature of all Hodgkin’s Lymphomas. hereditary spherocytosis Ref. Hemolysis and vaso-occlusive crisis are common in this form of anemia caused by mutation of glutamic acid to valine at amino acid 6 of beta chain. increased number of goblet cells B. Page 686 RESPIRATORY SYSTEM 57. Page 749 58. hemoglobin C C. A. Pulmonary emboli most often originate from which site? A. chronic necrotizing inflammation th Ref. Nodular sclerosis B. lipoma B. Page 742 17 . Pages 687-688 56.CARDIOVASCULAR SYSTEM 50. Page 584 51. destruction of alveolar walls D. Robbins 7th ed. Which of the following may have caused his symptoms? A. Page 629 54. Robbins 7 ed. mitral valve prolapse th Ref. mitral stenosis B. B-cells D. Page 594 52. Langhans giant cells th Ref. lobar pneumonia B. Pages1017-1018 65. steroid ulcers Ref. Neuroblastoma th Ref. omphalocele D. adenocarcinoma B. Pheochromocytoma D. Meckel’s diverticula B. A. hypertension C. Shatzki’s rings D. This is the most frequent primary lung tumor in men and associated with PTH secretion. embolism D. the tumor cells are large. Page 231 63. A. Robbins 7 ed. fever and abdominal pain during emotional stress. diffuse proliferative Ref. Page 744 KIDNEY AND URINARY SYSTEM 62. decubitus ulcers B. diffuse thickening of GBM on electron microscopy Ref. urine culture for resistant E. blastema and mesenchymal tissues. A.60. granular lumpy bumpy deposits of IgG C. small cell carcinoma Ref. Pages 504-506 64. What is the most likely diagnosis? A. These are linear lacerations occurring along the gastroesophageal junction due to fits of vomiting in alcoholics. A. Examination of the GI tract reveal longitudinal mucosal ulcerations and fistulas with intervening unaffected 18 . A 55 year old male presenting with hematuria. Which of the following tests is most valuable in confirming Goodpasture’s syndrome? A. This is a solitary congenital malformation in the small bowel due to persistence of the omphalomesenteric duct. mesoblastic nephroma D. Robbins 7th ed. A 2 year old boy presented with a large well circumscribed abdominal mass involving both kidneys. Oncocytoma B. Renal cell carcinoma B. Robbins 7th ed. with optically clear cytoplasm. no lesions B. flank pain and abdominal mass. Gross findings show a golden yellow tumor with hemorrhage and necrosis. intussusception C. esophageal varices B. LIVER. Atherosclerotic changes in the pulmonary arteries are generally indicative of this condition. Pages 819-820 68. Curling’s ulcers D. Robbins 7th ed. Mallory-Weiss tears C. These are diagnostic of which of the following? A. Page 830 69. Robbins 7 ed. Page 760 61. Robbins 7th ed. A. Microscopically. Adenoma Ref. A 30 year old Scandinavian often experiences intermittent attacks of diarrhea. Robbins 7th ed. Page 975 GIT. squamous cell carcinoma D. These are gastric ulcers associated with extensive burns. carcinoid C. Most common WHO class of Lupus Nephritis. Wilm’s Tumor C. Microscopic examination reveals epithelial tissues. diffuse linear staining of GBM with IgG B. mesangial C. drug abuse B. Page 802 67. Robbins 7th ed. PANCREAS 66. A. amyloidosis Ref. Wilm’s tumor C. Zenker’s diverticula th Ref. peptic ulcers C.coli D. Robbins 7th ed. focal segmental D. duplication Ref. Page 894 71. tubular C. Most common type of germ cell tumor in men of 3rd decade. Robbins 7th ed. A. Hepatitis C D. periampullary tumors B. mucinous Ref. What is the most likely diagnosis? A. Robbins 7th ed. weight loss and pre-tibial myxedema. These comprise majority of all invasive ductal carcinomas. classic B. atrophic endometrium D. Which of the following conditions are characterized by presence of endometrial glands and stroma outside the uterus? A. intraductal papilloma Ref. Pages 847-848 70. Page 1041 73. yolk sac tumor Ref. Acute thyroiditis B. complete mole C. This is the most common benign tumor of the breast. Tropical sprue C. Page 1149 76. Biopsy showed transmural inflammation and noncaseating granulomas. embryonal carcinoma C. Hashimoto’s thyroiditis C. choriocarcinoma D. What is the most likely diagnosis? A. Page 942 MALE AND FEMALE REPRODUCTIVE SYSTEM 72. seminoma B. Hepatitis A B. Ulcerative colitis Ref. fibroadenoma C. A 20 year old female presented with muscle weakness. adenomyosis B. Robbins 7th ed. Robbins 7th ed. diffuse hyperplasia 19 . invasive mole Ref. Grave’s disease Ref. Hepatitis D Ref. papillae and psammoma bodies. thiazide use C. Crohn’s disease D. Which of the following is one of the most frequent causes of acute pancreatitis? A. fibrocystic disease B. blunt duct adenosis D. Pages 1172-1173 78. This is a highly malignant neoplasm composed of a dual cell population of cytotrophoblasts and syncytiotrophoblasts. A. A. Typhoid ileitis B. seminoma B.segments. alcoholism Ref. Hepatitis B C. Page 1113 BREAST 75. What is the diagnosis? A. follicular carcinoma C. papillary carcinoma B. De Quervain’s thyroiditis C. Robbins 7th ed. A. Robbins 7th ed. Page 1083 74. teratoma D. medullary carcinoma D. Robbins 7th ed. Pages 1142-1143 ENDOCRINE SYSTEM 77. Robbins 7th ed. Anti-TSH receptors were positive. endometrial hyperplasia Ref. Examination of the thyroid show a 3 fold increase in size. Robbins 7th ed. cribriform D. Most common cause of transfusion-related hepatitis. trauma D. Fine needle aspirates of a 40 year old female showed atypical cells with nuclear grooves and inclusions. A. endometriosis C. basal cell carcinoma D. with separation artifacts and stromal mucin. Page 1160 MUSCULOSKELETAL SYSTEM 83. nodular D. What is targeted by the antibody detected? A. A. adenocarcinoma C. Page 1344 SKIN 85. Robbins 7th ed. Robbins 7th ed. A. they are characterized by peripheral palisading of tumor cells. Stevens-Johnson syndrome C. glomerular basement membranes C. prolactinoma B. myelolipoma Ref. TSH Ref.Ref. Sheehan’s syndrome D. This condition follows sudden infarction of the pituitary gland after obstetric hemorrhage or shock. Urticaria B. Myasthenia gravis is an autoimmune disease with increasing muscle fatigue with use and ocular muscle involvement. Pages 1234-1235 87. acral-lentigenous Ref. Pages 1242-1244 86. Most common form of malignant melanoma. A. A. These are adrenal masses which induces mark hypertension secondary to catecholamine production. growth hormone C. Robbins 7th ed. Microscopically. Robbins 7th ed. Robbins 7th ed. neuroblastoma B. This is the most common soft tissue tumor of adulthood. conjunctivitis and high fever. Page 1178 79. Follicular adenoma D. empty sella syndrome B. lipoma D. Lupus erythematosus Ref. squamous cell carcinoma B. Paget’s disease C. follicular carcinoma Ref. A. acetylcholine receptors B. This is the most common secreting tumor of the pituitary gland. A. xeroderma pigmentosum D. smooth muscle receptors Ref. Page 1317 84. histiocytoma Ref. Pages 1255-1256 20 . trichilemmoma Ref. lipoblastoma C. superficial spreading C. Robbins 7th ed. hibernoma B. What is the most probable diagnosis? A. Severe form of erythema multiforme with mucosal involvement. medullary carcinoma B. Robbins 7th ed. lentigo maligna B. craniopharyngioma C. pheochromocytoma D. thyroid stimulating hormone D. This is a skin lesion presenting as pearly papules often containing prominent dilated subepidermal blood vessels. Robbins 7th ed. papillary carcinoma C. A. Robbins 7th ed. Robbins 7th ed. Page 1163 82. ACTH D. Pages 1219-1221 81. This is the most common thyroid malignancy. Page 1178 80. lymphocytic hypophysitis Ref. p 137) Category: Recall 4. 7 th ed. 7 th ed. Which of the following is the most common cause of cerebrovascular accidents? A. This is the most common salivary gland neoplasm. A. enzymatic fat necrosis B. epithelial and lymphoid elements D. cell-mediated necrosis C. Pages 792-793 NERVOUS SYSTEM 91. A.. Oncocytoma C. embolic C. hypertrophy Answer: B (Robbins and Cotran Pathologic Basis of Disease. Adenoid cystic carcinoma B. Page 793 90. Robbins 7th ed. Mucoepidermoid carcinoma C. chondromyxoid matrix B. glioblastoma multiforme C. anaplasia D. Warthin tumor B. Pleomorphic adenoma Ref. Chapter 4. caseation necrosis Answer: D (Robbins and Cotran Pathologic Basis of Disease.HEAD. An increase of tissue volume due to the addition of new cells is: A. intracerebral hemorrhage D. 7 th ed. NECK AND EYE 88. Chapter 1. atherosclerosis B. The type of necrosis seen in tissue injury associated with acute pancreatitis is: A. This is the most common CNS tumor. A. inflammation D. Basal cell adenoma D.. Chapter 1 p 6) Category: Recall 3. passive congestion C. Which of the following is characteristic of a Warthin tumor? A. hyperplasia C. subarachnoid hemorrhage Ref. Robbins 7th ed. Tissue necrosis following occlusion of its blood supply is called: A. p 22) Category: Recall ACUTE AND CHRONIC INFLAMMATION 21 . Robbins 7th ed. infarction B. Robbins 7th ed. coagulation necrosis E.. metaplasia B. ependymoma Ref. oligodendroglioma D. Page 1363 CELL INJURY AND ADAPTATION 2. Page 791 89. Page 1401 100. traumatic necrosis D. Warthin tumor D. thrombosis Answer: C (Robbins and Cotran Pathologic Basis of Disease. This is the most common primary malignant salivary gland tumor. sheets of cells with clear cytoplasm C. Robbins 7th ed. hyaline material in between spaces Ref. Acinic cell tumor Ref. astrocytoma B. endothelial cells C.107) Category: Recall HEMODYNAMIC DISORDERS 12. Chapter 4 P 120) Category: Recall 22 . monocyte E. Chapter 3. epithelial cells D. 7 th ed. This largely is a function of: A. The hallmark of tissue healing is: A. Chapter 2.) Category: Recall 6. none of the above C. monocytes E. epithelioid cell D.. bone E. Chapter 3. 7 th ed.. Chapter 3. Chapter 1 p 179. 7th ed..79) Category: Recall TISSUE REPAIR 9. liver parenchyma C. basophils Answer: D (Robbins and Cotran Pathologic Basis of Disease.) Category: Recall 10..) Category: Recall 7.. neutrophils D. p. Chapter 1 p 82. The hallmark of granulomatous inflammation is the: A. fibroblasts Answer: C (Robbins and Cotran Pathologic Basis of Disease. lymphocytes Answer: B (Robbins and Cotran Pathologic Basis of Disease. 7th ed. myocardium D. lymphocytes B. increase vascular permeability E. low serum albumin D. 7th ed. neutrophil Answer: A (Robbins and Cotran Pathologic Basis of Disease. plasma cells C. neutrophils E. plasma cell D. Macrophages are invariably found in abundance in inflammatory reactions. p. Chapter 1 p 57) Category: Recall 8.. p9091. exudation D. decrease vascular permeability Answer: D (Robbins and Cotran Pathologic Basis of Disease. In the course of cellular repair. The main pathogenetic mechanism of edema in inflammatory diseases is: A. 7 th ed. granulation tissue C. decrease hydrostatic intravascular pressure C.) Category: Recall 11. increase osmotic pressure B. The type of cell that is first to migrate into foci of acute inflammation is the: A. 7 th ed.. granuloma B. eosinophils D. One of the following is incapable of postnatal mitosis: A. lymphocyte infiltration Answer: A (Robbins and Cotran Pathologic Basis of Disease. skin epithelium Answer: A (Robbins and Cotran Pathologic Basis of Disease. astrocytes E. malignant lymphocytes B.5.. fibroblast B. Chronic inflammation most often contains: A. p110. giant cell C. lymphocyte Answer: D (Robbins and Cotran Pathologic Basis of Disease. fibroblast B. These are derived from the: A. plasma cell E. endothelial cells B. fibroblast C. polymorphonuclear leukocyte B. 7 th ed. there often is collagen deposition. 13. Sudden death in pulmonary embolism is due to: A. acute right heart failure D. superior vena cava syndrome B. cardiac tamponade E. respiratory distress syndrome C. pulmonary edema Answer: A (Robbins and Cotran Pathologic Basis of Disease, 7th ed.; Chapter 4 p 136) Category: Application 15. Localized edema is best exemplified by: A. low serum protein B. nephritic syndrome C. portal hypertension Answer: D Category: Recall D. lymphatic obstruction E. congestive heart failure DISEASES OF IMMUNITY 16. Tumor most commonly associated with AIDS: A. adenocarcinoma D. melanoma B. hepatoma E. lymphoma C. Kaposi’s sarcoma Answer: C (Robbins and Cotran Pathologic Basis of Disease, 7 th ed.; Chapter 6 p 256) Category: Recall 17. Systemic Lupus Erythematosus is a variable multi-organ disease, but in 100% of cases lesions always occur in the: A. brain C. spleen B. liver D. kidney Answer: D (Robbins and Cotran Pathologic Basis of Disease, 7th ed.; Chapter 6 p 231) Category: Application 18. Pneumonia caused by this organism is frequently the first diagnosed infection in HIV-infected persons: A. Pneumocytstis carinii C. Cryptococcus neoformans B. Aspergilus sp. D. Histoplasma capsulatum Answer: A (Robbins and Cotran Pathologic Basis of Disease, 7 th ed.; Chapter 15,, p 765.) Category: Recall PRINCIPLES OF NEOPLASIA 21. The most frequent form of cancer in females: A. gastric D. breast B. uterine E. lung C. colonic Answer: D (Robbins and Cotran Pathologic Basis of Disease, 7 th ed.; Chapter 7 p 282.) Category: Recall 22. The most common cause of death due to cancer in both men and women is: A. malignant lymphoma C. lung carcinoma B. colorectal carcinoma D. hepatocellular carcinoma Answer: C (Robbins and Cotran Pathologic Basis of Disease, 7 th ed.; Chapter 7, p 282.) Category: Recall INFECTIOUS DISEASES 29. TB and Schistosoma can produce: A. ulceration D. cavitation B. caseation E. granuloma C. meningitis Answer: E (Robbins and Cotran Pathologic Basis of Disease, 7 th ed.; Chapter 8, p 384, 409) 23 Category: Recall MULTIPLE CHOICE: 1. The hallmark of acute inflammation is: A. Increased blood flow B. Rubor, calor, dolor increased permeability of microcirculation C. Increased vascular permeability D. Vascular stasis 2. The most efficient bactericidal compound present in neutrophils is the: A. Arachidonic acid metabolites C. Lysosomal enzymes B. H202-MPO-Halide system D. Hydrogen peroxide 3. Leukotrienes are derived from arachidonic acid through the help of this enzyme: A. Phospholipase C. Cyclooxygenase B. Lipooxygenase D. Lipoxins 4. The hallmark of chronic inflammation is: A. Increased permeability of microcirculation B. Migration of macrophages at site of injury C. Tissue destruction D. Mononuclear cell infiltration 5. An example of tumor suppressor gene: A. p53 B. C-myc C. ras D. bcr 6. A lymph node biopsy was diagnosed as tuberculosis. This type of necrosis is characteristic of: A. Coagulative C. Caseous B. Liquefactive D. Gangrenous 7. Which of the following substances accumulates in atherosclerotic blood vessels? A. Fatty acids C. Tryglicerides B. Cholesterol esters D. Phospholipids 8. "Heart failure cells" found in the lungs in Chronic Passive Congestion are actually…. A. Lymphocytes C. Neutrophils B. Eosinophils D. Macrophages 9. The outcome of thrombosis wherein the thrombi tends to GROW due to deposition of additional platelets, fibrin and red blood cells: A. Resolution C. Propagation B. Organization D. Recanalization 10. Most common cause of death in burn patients who got into shock is: A. Sepsis C. Dehydration B. CHF D. Hemorrhage 11. This type of hypersensitivity reaction is mediated by IgE: A. Type I C. Type III B. Type II D. Type IV 12. One of the three major components of acute inflammation include: A. Alteration in vascular caliber that leads to a decrease in blood flow B. Structure changes in the microvasculature permitting plasma proteins and leukocytes to leave the circulation C. Tissue destruction D. Healing by connective tissue 24 13. Heat and redness in acute inflammation is due to the following pathogenetic mechanism: A. Increased vascular permeability C. Stasis B. Increased blood flow D. Leukocytic margination 14. The uterine endometrial lining thickens during the proliferative phase of the menstrual cycle. This is due to: A. Hypertrophy C. Hyperplasia B. Dysplasia D. Metaplasia 15. Which of the following disorders is most likely to be associated with an exudates rather than a transudate? A. Congestive heart failure C. Suppurative inflammation B. Nephrotic syndrome D. Peritonitis 17. The role of histamine in acute inflammatory response include: A. Platelet release and aggregation B. Increased vascular permeability of the venules C. Increased vascular permeability of the arterioles D. Membrane lysis 18. The central figure in chronic inflammation is the Macrophage because of its role in: A. Breakdown of collagen and fibronectin B. Production of 02 and Nitric oxide metabolites C. Emigration from the blood stream to site of injury D. Inhibition by cytokines and oxidized lipids 19. A biopsy of the stomach reveal an area of malignant change characterized by varisized, compactly arranged neoplastic glands lined by moderately differentiated cells. Which of the following terms refer to malignant tumor of glandular epithelium? A. Sarcoma C. Teratoma B. Choristoma D. Adenocarcinoma 20. Which of the following statements in NOT true of red infarcts? A. It occurs in organs with double blood supply B. Commonly found in the heart and kidneys C. Seen in venous occlusion with infarction D. Seen in organs with well developed anastomosis 21. Which of the following conditions is a complication of both direct injuries to the lungs and systemic disorders? A. Atelectasis C. Diffuse alveolar damage B. Pulmonary edema D. Pulmonary congestion 22. Which of the following conditions of the lung is characterized by abnormal permanent enlargement of the air space distal to the terminal bronchiole accompanied by destruction of their walls without obvious fibrosis? A. Emphysema C. Chronic bronchitis B. Bronchial asthma D. Bronchiectasis 23. A spectrum of immunologically mediated predominantly interstitial lung disorders caused by intense often prolonged exposure to inhaled organic dusts and occupational antigens is: A. Pulmonary alveolar proteinosis B. Bronchiolis obliterans-organizing pneumonia C. Desquamative interstitial pneumonitis D. Hypersensitivity pneumonitis 24. An acute febrile respiratory disease characterized by patchy inflammatory changes in the lungs confined to the alveolar septa and pulmonary interstitium: A. Primary atypical pneumonia C. Bronchopneumonia B. Lobar pneumonia D. Lobular bronchopneumonia 25 Plaques D. What is the most likely diagnosis? A. 60-year-old man. Non-atopic asthma C. Sarcoidosis D. He had one bout of hemoptysis and went to the ER for consultation. Most common type of asthma that begins in childhood and triggered by environmental antigens such as dust. Presence of malignant squamous cells in sputum C. Most common benign tumor of the lung discovered incidentally as a coin lesion on routine X-Ray examination is: A. Bronchiectasis D. X-ray shows a 6 cm mass on the medial upper lobe. Bronchogenic carcinoma C. Hilar area C. Hamartoma B. he experienced chronic cough and weight loss for the past 6 months. Idiopathic pulmonary fibrosis B. Presence of acid fast organism on sputum exam B. A fatal complication of ruptured aortic aneurysm and vascular trauma: A. no difficulty of breathing. Hydrothorax B. Bronchial carcinoids C. Which of the following cytologic findings is likely to be found in this patient? A. Ferruginous bodies B. no fever. Metastatic carcinoma D.25. no fever no nausea and vomiting. Chylothorax C. and skin is: A. lungs. Bronchopulmonary sequestration C. Most common location of secondary tuberculosis in the lung: A. Base D. Asbestos body C. Atelectasis B. Drug induced asthma d. Mang Juanito. 55-year-old man has no major medical problems in the past year. A systemic disease of unknown cause characterized by non-caseating granuloma in many tissues and organs particularly in the hilar lymph nodes. eye. 4 months prior to consultation the patient experience malaise and weight loss of 10 kg. Malignant mesothelioma B. Goodpasture syndrome C. Empyema 33. Amphibole 30. Silicosis C. Bronchioalveolar carcinoma 36. Peripheral B. Karteagner syndrome B. Asbestosis 26. Apex 32. Chest X-ray shows a multiple solid nodules scattered throughout the lung fields. Most common manifestation of asbestos exposure often containing calcium seen on the anterior and posterior aspect of the parietal pleura: A. Bronchoscopy shows a mass on the segmental bronchus. Upon intubation he suffers aspiration of gastric contents. Hemothorax D. Staphylococcus aureus D. Paraneoplastic syndrome D. Presence of numerous necrotic debris and inflammatory cells in sputum D. Atopic asthma B. pollens and food is: A. Which of the following syndrome is characterized by the appearance of proliferative glomerulonephritis and necrotizing hemorrhagic interstitial pneumonitis? A. The patient is a non-smoker. Chest X-ray shows a 3 cm mass in the right lung with elevated air fluid level. 45-year-old man had an episode of myocardial infarction he was brought to the emergency room and was intubated. Bronchogenic cyst 27. Mang Tomas. Presence of reactive mesothelial cells in pleural fluid exam 35. Lung abscess 26 . Occupational Asthma 29. Haemophilus influenzae B. Mang Tony. Thymoma D. Pseudomonas aeroginosa 28. has been smoking for many years. For the next 5 days he develops non-reproductive cough and fever. Lambert-Eaton myasthenic syndrome 31. What is the most likely diagnosis? A. Streptococcus pneumonia C. Most common cause of aspiration pneumonia is hospitalized patients: A. 9 mg/dl and a direct bilirubin level of 6. inflammation is limited to the: A. Decreased interstitial osmotic pressure D. A lymphohematogenous dissemination of pulmonary tuberculosis would give rise to: A. The serum Alanine aminotrasferase (ALT) level is 27 . Large areas of hemorrhagic green ulceration of the mucosa and green-black necrosis through the wall. Neutrophils 45. Hemodynamic pulmonary edema seen in congestive heart failure is due to: A. 40-year-old female. Increased hydrostatic pressure B. Which of the following conditions best accounts for these findings? A. Crohn's disease B. Tuberculous bronchopneumonia B. Defective Intraluminal digestion B. male suffers from episodic abdominal bloating with flatulence and explosive diarrhea after attending a week-long community celebration of the dairy industry during the rest of the year. Acute suppurative appendicitis B. Serosa B. Josie. Blockage of blood flow through the lungs B. he does not consume milk shakes or ice cream and is not symptomatic. Development of pulmonary hypertension 39.8 mg/dl. Increased oncotic pressure C. Tropical sprue D. Transmural layer 47. Mucosa C. In ulcerative colitis. Celiac sprue C. Celiac sprue C. The most likely diagnosis is: A. Miliary tuberculosis 40. Potts disease D. Development of chronic obstructive pulmonary disease D. Disaccharidase deficiency B. Disaccharidase deficiency is classified under which of the following major causes of malabsorption syndrome: A. Increase responsiveness of the bronchial tree 41. Atelectasis C. Microscopic findings of the lesion showed inflammation from the mucosa to the serosa with the presence of non-caseating granulomas. The plausible mechanism in the development of emphysema is: A. Reduced small intestinal surface area 46. Histiocytes C. Presence of obstruction of a tumor of foreign body with concomitant infection D. Vacitary fibrocaseous tuberculosis C. Deficiency of Vitamin B12 C. Henry. has a total serum bilirubin concentration of 8. The histologic criterion for the diagnosis of acute appendicitis is based on finding this type of cell infiltrating the muscularis propia: A. Increase alpha-1 anti-trypsin enzyme C. 25-year-old male complained of intermittent diarrhea and lower abdominal pain. Lymphatic obstruction 43. 45-year-old. Hypersensitivity to gliadin fraction of wheat gluten D. Ulcerative colitis D. Bacterial infection B. The major cause of celiac sprue is: A. Acute gangrenous appendicitis C. Lymphocytes B. Sudden death in patients with pulmonary embolism is caused by: A. Lymphatic obstruction C. Submucosa D. Leon. An upper G-I series showed segmental narrowing in the ileum. Whipple's disease 50. extending to the serosa of the appendix is most likely associated to which of the following stages of acute appendicitis? A. Primary mucosal cell abnormalities D. Whipple's disease 49. Increase release of elastase in neutrophils and macrophages B. Ruptured acute appendicitis 48. Monocytes D.37. Increased albumin 38. Early acute appendicitis D. Lymphoid hyperplasia and multinucleated giant cells within the muscular wall D. reddish brown rash and was treated with aspirin. Nelia. Which of the following findings is more compatible to Crohn's Disease than to ulcerative colitis? A. An appendix with normal appearance B. Hepatocellular tumor 57. girl. Imelda. 45-year-old. 25-year-old. female. Alcoholic cirrhosis D. An upper G. and serum antimitochondrial autobodies are present. and an elevation of the peripheral leukocyte count to 17x109/L. 25-year-old. medical student developed sharp epigastric pain relieved by eating. Reflux esophagitis D. and the aspartate aminotransferase (AST) level is 108 U/L. B. series demonstrated a 1 cm gastric ulcer. and rectal bleeding. Ulcerative lesion in the greater curvature is more likely to be classic peptic ulcer 53. Poliomyelitis 56. She is found to have high serum alkaline phosphatase and slightly elevated serum bilirubin levels. Ulcerative colitis D. Tropical sprue B. Pseudopolyp formation between ulcers D. The base of the peptic ulcer is smooth and clean D. He was admitted to the emergency room because of lobar pneumonia. Crypt abscess formation deep in mucosa B. Monometry reveals a peristalsis.125 U/L. dyspnea. Sigmoidoscopy showed numerous ulcers. Alcoholic hepatitis C. He was found comatose and in liver failure. Joan. Joshua. Increase incidence of adenocarcinoma of colon C. A liver biopsy reveals a marked Lymphocytic infiltrate in the portal tracts. Idiopathic inflammatory bowel disease was considered. Esophageal diverticulum 55. A liver biopsy shows histologic findings characteristic for Sclerosing cholangitis. Ronnie. Whipple's disease 51. Celiac sprue C. I. The 28 . 38-year-old. This finding is most typical of: A. Hepatic steatosis B. Reye's syndrome D. Varicella-Zoster infection C. Examination of the surgically resected appendix is most likely to reveal: A. Neutrophils within the muscular wall C. regurgitation of sour brash. Which of the following gastrointestinal tract disease is most likely to coexist in this patient? A. Sliding hiatal hernia B. Presence of granulomas in the colonic wall 54. 6-year-old. The most likely diagnosis is: A. presents with abdominal pain localized to the right lower quadrant. Subacute Sclerosing panencephalitis B. she developed fatty change of the liver. Heaping-up margin is rare in the benign ulcer but characteristic of malignant lesion C. Occasional granulomas are also seen. Fely. Cesar. Zenker's diverticulum 52. complains of fatigue and pruritus. 18-year-old. 35-year-old male had a history of heart burn. female. mild fever. female. Which of the following features is not compatible with the clinical findings? A. Esophageal atresia C. nausea and vomiting. The size of the ulcer does not differentiate a benign from malignant ulcer. had a blocky. alcoholic went on a binge for 2 weeks. Achalasia D. His liver at autopsy showed several hepatocytes with hyaline Mallory bodies in the cytoplasm. A dilated lumen filled with mucus 58. The most possible cause of his pneumonia is due to aspiration and can be attributed to which of the following conditions? A. Stenosis due to severe gastroesophageal reflux B. abdominal pain. The most possible diagnosis is: A. burning esophageal pain and slowly but progressive dysphagia. Paraesophageal hiatal hernia C. A middle-aged male complains of food sticking somewhere between mouth and stomach. developed diarrhea. student. Rhabdomyoma B. VSD C. Polymorphonuclear cells B. Primary Sclerosing cholangitis B. Hyperlipidemia syndromes 60. Coarctation B. 15-year-old. It is normal finding 71. Diphtheria 69. Serous pericarditis B. Except: A. Angina that is unstable is clinically described as: A. Impacted gallstone 59. Lungs D. Cases of patent ductus arteriosus that are not isolated are commonly associated with the following conditions. What is the diagnosis? A. Brain 65. Leiomyoma C. This organ is not prominent affected in right-sided failure: A. How do you interpret this? A. Kidneys B. Liver C. A cardiac mass form an infant on microscopic examination revealed "spider cells". Eosinophils 63. Purulent pericarditis D. Patient had NBTE B. died of sepsis with DIC. George. Histopathology examination of the heart of a patient who died of AMI showed coagulative necrosis. Myxoma D.most likely diagnosis is: A. edema. hemorrhage and neutrophilic infiltrates. had sore throat and a week later developed swollen joints and a murmur in the aortic valve area. Daniel. hard round stones within the gallbladder is NOT associated with which of the following? A. The bile salts aid in the emulsification of dietary fats in the intestines. This is due to: A. Left circumflex D. 40-year-old male. Dilated sinusoids and irregular cystic spaces filled with blood within the liver which may rupture leading to massive intra-abdominal hemorrhage. Macrophages D. Left anterior descending C. pale. small masses of fibrin thrombi were seen in the heart valves. Transient C. Alveolar lining cells C. The LEAST affected one is the: A. Fibrinous pericarditis 70. SLE B. Precipitated by physical stress B. Biliary infection D. yellow. Infective endocarditis C. Primary biliary cirrhosis C. The findings of multiple. Another infarct C. are most commonly associated with: A. What type of tumor is this? A. Obesity B. Fibroma 66. 30 minutes C. 1 week 68. who recently had an infarct developed a loud pericardial friction rub. Viral hepatitis B infection D. Acetaminophen 61. Rheumatic heart disease D. Transmural infarcts commonly involve these three arterial vessels of the heart. Noli. Estrogen B. Right circumflex 62. Progressive 64. The heat failure cells are by nature: A. Albumin C. Patient had rheumatic heart disease D. They are composed of bile acids that have been conjugated with: A. 35-year-old man. How old is the infarct? A. Glucoronic acid 29 . Oral contraceptive C. 1 day B. Upon autopsy. Anabolic steroids D. Right coronary B. ASD D. Pulmonary stenosis 67. Salicylates C. Relieved by rest D. 12 hours D. Patient had infective endocarditis C. The lipemic appearance is caused by the presence of: A. heart and brain is grossly active muscular dystrophy and rises early in myocardial infarction is: A. Glucose 72. Right sided failure may result in hepatomegaly B.lipoprotein fraction increased 4. Which type of inflammation is most characteristic of acute rheumatic fever? A.B. Should be counseled to modify his diet to reduce his total cholesterol C. Is probably not borderline high risk for coronary heart disease. Based on these results. Myocarditis C.235 mg/dl (NV 40-164 mg/dl) 2. Tangier D. The milky appearance of serum after fat ingestion is called post prandial lipemia. The test results obtained are: 1. The enzyme that exists chiefly in skeletal muscle. Pericarditis D. Lipase C. Four beta chains C. Total cholesterol . Creatine kinase 79.190 mg/dl (NV less than 200 mg/dl) 3. Howell-Jolly bodies are composed of: A. Type IV hyperlipoproteinemia 81. Mitochondria 83. Prebeta . Dyspnea is a result of blood stasis in the extremities 77. The clinical manifestation of heart failure most commonly reflect right-sided failure D. Patent ductus arteriosus B. Should be consulted to see his physician immediate to follow-up testing D. when present in an increased concentration. Four alpha chains B. Four gamma chains D. Juanito. Chylomicrons D. Krabbe's C. Serum appearance milky The best explanation for these results would be that the individual exhibited characteristic of: A. Lactate dehydrogenase D. The most mature cell that can undergo mitosis is the: 30 . Which of the following diseases results from a familial absence of high-density lipoprotein? A. A blood specimen is drawn for lipoprotein phenotyping. This disease of the heart presents on early cyanosis: A. Fatty acids B. Triglycerides . two beta chains 82. Is a borderline high risk of coronary heart disease B. Iron D. Apo A-I C. Phospholipids 73. Hemoglobin Bart's is composed of: A. Cholesterol C. Atrial septal defect C. 80. RNA C. Truncus arteriosus D. Apo B-48 D. Type II hyperlipoproteinemia B. Transminase B. Pancarditis B. DNA B. Two alpha. would be associated with a decreased risk of coronary artery disease? A. Cor pulmonale usually is due to severe pulmonic stenosis C. Apo C-II 74. Chylomicrons present 6. Amino acid D. Which of the following apolipoproteins.lipoprotein fraction normal 5. Gaucher's B. A non-fasting serum protein D. 34-year-old was found to have total cholesterol of 225 mg/dl and an HDL cholesterol of 83 mg/dl. A normal individual C. Tay-Sachs 78. this individual: A. Which of the following statements about heart failure is true? A. Beta. Endocarditis 75. Apo B-100 B. Atrioventricular septal defect 76. Bone marrow D. and fetal hemoglobin D. Spleen B. hemoglobin structure 92. In early infancy the most numerous cells of the bone marrow are: A. A decreased in which of the following laboratory results is NOT a usual diagnostic criterion for anemia? A. Mega karyoblasts C. Neutrophils 87. Lymphocyte B. Methemoglobin and fetal hemoglobin C. Which of the following is not a crucial area of RBC survival and function? A. Lymphocytes that produce immunoglobulins in response to antigenic stimulation are designated: A. RBC count 95. Lymphocytes D. Thymocytes 86. Myeloblast B. Carboxyhemoglobin. Plasmacyte 90. Kidney 88. Metamyelocyte 94. Which of the following groups of abnormal hemoglobins are unable to transport or deliver oxygen? A. Promyelocyte D. Myelocyte 84. usually small and round. Intravascular hemolysis D. sulfhemoglobin. Basophils B. Gaucher's cells 31 . Determines response and potential of the bone marrow B. Granulocytic precursors B. Iron stage 91. Eosinophils D. Carboxyhemoglobin. hematocrit B. Cell metabolism B. B Lymphocytes C. What is diagnostic value of reticulocyte count in the evaluation of anemia? A. Carboxyhemoglobin and methemoglobin B. M:E ratio B. Myelocyte C. Eosinophilic C. and nucleus with clumped dark purple chromatin? A. The cytoplasm of these cell stains very pale and has a striated appearance: A. Metamyelocyte D. Histiocytes-monocytes 89. Monocyte C. Erythroblasts C. Plasma cells B. Null cell D. intensely blue cytoplasm. A bone marrow has large cells that have eccentric pyknotic nuclei. Hemoglobin C. Integrity of RBC cellular membrane C. Which cell classification is described by the following: Second most numerous cell in the blood. Monocytes B. Differential count of 500 cells C. Lymphocytes C. methemoglobin and sulfhemoglobin 93. Which of the following organs is NOT a site for hematopoiesis in the fetus? A. Determines compensation mechanisms for anemia C. Cellularity D. T lymphocytes D. Myeloblast B. Liver C. Platelet count D. Production of primary granules ceases and production of secondary granules commences with what cell stage? A. Toxic granulation is most commonly observed as a Cytoplasmic inclusion of: A. Determines the corrected RBC count after the calculation D. Determines the potential sampling error for RBC count 96. Promyelocyte C.A. Monocytes 85. Neutrophils D. Which of the following is most variable in normal marrow? A. Vasodilation and bronchoconstriction would be associated with which blood cell: A. Medullar thyroid carcinoma B. Medullary carcinoma B. parathyroid hypoplasia th Robbin’s 7 Edition Chapter 24. Page 1166 2. Maturation of red and white blood cells series B. ENDOCRINE PATHOLOGY 1. Page 1177 5. parathyroid carcinoma B. Which of the following is NOT a factor to be evaluated in the interpretation of a bone marrow aspirate smear? A. CHOOSE THE BEST ANSWER. Phagocytosis B. A patient with tremors and with exopthalmos has enlarged thyroid gland. Riedels thyroiditis D. A medical student who suddenly developed marked tetany and parasthesia and later on was found out that she was suffering from a parathyroid problem.Annie nuclei is a characteristic nuclear feature of this thyroid malignancy 32 . Papillary carcinoma C. Hashimotos thyroiditis C. 56) Lypmhocytes . Your diagnosis would be: A. Granulomatous thyroidis B. Anaplastic thyroid carcinoma Robbin’s 7th Edition Chapter 24. Follicular carcinoma D. Lito 27-year-old has a total WBC count of 4x109/L. Chemotaxis C.2-) Monocytes .34) Band . fixed thyroid gland associated with extensive fibrosis that is contiguous with adjacent neck structures A. If thyroidectomy is done the thyroid would exhibit this type of cellular adaptation. The percentage of lymphocytes is normal B..28 (NV . There is an absolute lymphocytosis D.. Which of the following thyroid malignancy has the highest mortality? A. Hypertrophy D. Graves disease Robbin’s 7th Edition Chapter 24.5 (NV . Reed-Sternberg cells D. Estimate of bone marrow activity 98. Page 1178 6. Type and amount of hemoglobin D.02 (NV . A. Microscopic sections of a 3 cm solitary thyroid nodule show proliferating fetal type of thyroid follicles with a focus a transcapsular invasion. Your diagnosis would be: A. Page 1171 3. Production of oxygen and radical Reference : Pathologic Basis of Disease 6 th Edition SIMPLE MULTIPLE CHOICE.65 (NV . Hyperplasia C. The principal defect in chronic granulomatous disease is in: A. The differential count is as follows: Neutrophils . Metaplasia B.. M:E ratio C. There is a relative lymphocytosis 100. parathyroid adenoma D. Follicular carcinoma D. Papillary carcinoma C. The absolute number of lymphocytes is low C. Page 1167 4. This is characterized by hard. Orphan. Large myeloblasts 97. Lysosomal function D. Anaplastic thyroid carcinoma Robbin’s 7th Edition Chapter 24. Dysplasia th Robbin’s 7 Edition Chapter 24.4) Which of the following statements is true? A.B. hypoparathyroidism C. Fraction diverticulum Robbin’s 7th Edition Chapter 17. Metastatic carcinoma B. An enlarged axillary lymph node excised from a 53 year old female presenting with an ill defined. Primary malignant tumor B. It is a common cause of gastric malignancy D. Page 279 12. tail D. fatty change B. Hepatoma C. The usual site of pancreatic malignancy is: A. You would think of a malignant ulcer if this gross feature is seen A. Cholangiocarcinoma D. Metaplasia B. Achalasia C. accessory pancreas 10. Esophageal varices B. Hypertrophy D. Head B. Metastatic tumor C. tuberculosis C. Samonella B. The type of ulcer produced by this organism is flask – shaped A. Page 802 13. Alpha feto protein is requested for the detection of this liver neoplasm A. Inflammatory polyp 14. This is an premalignant colonic polyp A. Barretts esophagus shows this adoptive cellular change A. pylori organisms aggregates along metaplastic epithelium B. Mallory weiss D. Papillary carcinoma B. Page 857 C. Medullary carcinoma D. Puetz Jegher polyp D. Page 813 15. Page 10 8. Crohns disease Robbin’s 7th Edition Chapter 17. H. Klatskin tumor Robbin’s 7th Edition Chapter 7. E. LIVER. Anaplstic carcinoma GIT. Hyperplasia C. Hamartomatous tumor Robbin’s 7th Edition Chapter 7. Mallory weiss D.A. bridging necrosis 9. Severe esophagitis Robbin’s 7th Edition Chapter 17. fibrosis D. Page 839 16. Page 800 17. Page 339 11. Dysplasia Robbin’s 7th Edition Chapter 1. Which is true of chronic gastritis? A. M. fixed palpable firm mass at the right upper outer quadrant would probably show A. It commonly presents as bleeding Robbin’s 7th Edition Chapter 2417. It is grossly seen as multiple ulceration C. Body th Robbin’s 7 Edition Chapter 19. edematous border 33 . Hamartomatous polyp B. converging mucosal fold C. Page 950 C. Malignant transformation is a common complication of this lesion A. Page 1178 C. Follicular carcinoma Robbin’s 7th Edition Chapter 24. cellular swelling Robbin’s 7th Edition Chapter 18. Benign tumor D. Page 881 C. histolytica D. AND HBT 7. This lesion is grossly seen as linear laceration in the gastroesophageal area A. The hallmark of irreversible liver injury A. Barretts esophagus C. Peptic ulcer B. Adenomatous polyp th Robbin’s 7 Edition Chapter 17. Lipofuscin D. Page 8 24. Page 383 26. Most common malignancy of the appendix is: A. Linitis plastica th Robbin’s 7 Edition Chapter 17. Page 317 34 . Cervical biopsy of a 32 year old female with cervical erosion shows squamous epithelium that exhibits some atypia and loss of polarity. fibrosis B. Most common location of gastric carcinoma A. Page 274 28. A. necrotic tissues D.B. Nuclear pyknosis C. Mycobacterium tuberculosis infection would show this type of necrosis. Page 12 NEOPLASIA 27. Dilatation of ER D. A gastric carcinoma that metastasize in the periumbillical area is called: A. Metaplasia B. Page 77 25. Bilirubin pigments Robbin’s 7th Edition Chapter 1. Virchows nodule C. APC D. malnutrition B. p-53 Robbin’s 7th Edition Chapter 7. Which of the following is an irreversible form of cellular injury? A. This adoptive response is: A. Squamous cell carcinoma D. mucosa overhungs the C. Page 822 C. portal vein thrombosis D. Gastric malignancy D. Caseation D. Liquefaction B. Hypertrophy D. The deepest layer of an ulcer is made up of which of these? A. Page 850 18. vessel proliferation C. Coagulation C. Page 825 20. Congenital megacolon 21. This is the most activated oncogene in adenomas and colon cancers: A. Stercoral ulcer are associated with this disease: A. Councilman bodies B. Fundus B. Meckels diverticulum B. Atrophy of myocardial fibers would show this cellular accumulation A. Lymphoma Robbin’s 7th Edition Chapter 17. Hemosiderophage C. Sister Mary Joseph nodule D. Antrum 22. uneven borders edge of the ulcer Robbin’s 7th Edition Chapter 17. Chronic gastritis Robbin’s 7th Edition Chapter 17. Bax gene C. Seminoma 19. K-ras B. Page 871 D. Gangrenous Robbin’s 7th Edition Chapter 8. Krukenberg tumor B. acute left ventricular failure C. Page 830 C. Hyperpalsia C. Cardia D. chronic right ventricular failure Robbin’s 7th Edition Chapter 4. Nutmeg liver is the gross appearance of the liver in: A. mixed inflammatory cells th Robbin’s 7 Edition Chapter 2. Dysplasia Robbin’s 7th Edition Chapter 7. Page 123 CELLULAR REACTION TO INJURY (CRI) 23. Carcinoid B. Body Robbin’s 7th Edition Chapter 17. Plasma membrane blebs th Robbin’s 7 Edition Chapter 1. Mitochondrial swelling B. Page 273 C. Adenomatous polyp D. sarcoma D. The tumor would be very pleomorphic D. CIN C. A.29. Aggregates of epitheloid histiocytes th Robbin’s 7 Edition Chapter 3. Contraction 35 . CLL B. structural and functional differentiation of cell is known as: A. Fibroepithelial polyp B. Absolute lymphocytosis. Identification of Reed-Sternberg cells will lead you to think of this tumor: A. Amount of Granulation tissue D. The tumor is severely anaplastic B. Page 281 35. Anaplasia D. carcinoma B. ALL C. AML D. Hodgkins lymphoma D. Page 272 34. The main difference between healing by primary intention and secondary intention is: A. The tumor still looks like the tissue where it originates C. adenoma Robbin’s 7th Edition Chapter 7. Page 271 C. dysplasia Robbin’s 7th Edition Chapter 7. hyperchromaticity B. A malignant epithelial tumor is referred to as: A. Non-hodgkins lymphoma C. anemia 31. Page 1032 INFLAMMATION 38. Loss of organizational. Sarcoma 36. Which of the following is a benign tumor ? A. lymphoma 33. Aniline dye and rubber can cause malignancy to this organ A. Page 667 C. many small mature lymphocytes in the peripheral smears is a characteristic of this type of leukemia: A. infection Robbin’s 7th Edition Chapter 14. A well differentiated tumor means: A. Leukocyte migration C. gout B. Presence of harphazardly arranged bundles of collagen D. Page 115 39. Which of the following is the pre-cancerous lesion A. Urinary bladder Robbin’s 7th Edition Chapter 21. lymphoblasts. CML Robbin’s 7th Edition Chapter 14. Neoplasia C. hairy cell leukemia th Robbin’s 7 Edition Chapter 14. Hemangioma D. Lymphoid follicle formation C. Liver D. Fibrosis B. diabetes D. Ulcerative colitis th Robbin’s 7 Edition Chapter 25. Hepatoma B. Burkitts lymphoma B. The tumor is benign Robbin’s 7th Edition Chapter 7. Page 670 30. This is not a common complication of leukemia: A. Angiogenesis B. Retinoblast Robbin’s 7th Edition Chapter 7. Page 1238 37. A hypertrophied scar would show this feature. Cervix C. Skin B. Page 686 32. Page 750 48. Page 79 RESPIRATORY SYSTEM 41. Oat cell carcinoma C. Congestion C. Cardiogenic shock D. Page 751 46. Small cell carcinoma th Robbin’s 7 Edition Chapter 15. Septic shock th Robbin’s 7 Edition Chapter 4. Hypovolemic shock B. Patchy atelectasis B. Klebsiella C. deficiency in pulmonary surfactant C. Asthma 36 . Page 139 42. This type of malignancy is commonly seen in the periphery of the lungs A. Page 799 44. Bronchiectasis C. pulmonary sequestration D. Strep pneumonia B. Incerase premeability of blood vessels Robbin’s 7th Edition Chapter 2. Gray hepatization D. Red hepatization Robbin’s 7th Edition Chapter 15. Posterior mediastinal rest of the lungs tissue with no connection to the air way system is known as: A. Page 760 49. Proliferation of blood vessels C. increased hydrostatic pressure D. Viral pneumonias typically presents in the lungs as: A. Compression atelectasis C. Staph aureus th Robbin’s 7 Edition Chapter 15. Page 715 43. Page 748 47. lymphoplasmacytic infiltrates within the alveolar sacs B. Page 727 45. This is the second stage in the evolution of the lobar pneumonia: A. Resolution B. Collapse of lung secondary to air in the pleural cavity is an example of: A. Emigration of neutrophils B. This lung disease exhibits smooth muscle hyperplasia of the bronchial walls A. Resorption atelectasis D. inflammatory reaction to microorganisms Robbin’s 7th Edition Chapter 15. Adenocarcinoma B. neutrophils in the peribronchiolar area Robbin’s 7th Edition Chapter 15. teratoma B. Pulmonary embolism could lead to this type of shock A. Emphysema C. Pulmonary sequestration B. This syndrome is characterized by bronchiectasis.Robbin’s 7th Edition Chapter 3. Squamous cell carcinoma D. Page 714 50. This process is always seen in chronic inflammation: A. The mechanism of adult respiratory distress syndrome is: A. Contraction atelectasis Robbin’s 7th Edition Chapter 15. accumulation of neutrophils in one lobar segment D. Increase in edema fluid D. Haemophilus influenza D. Mesothelioma th Robbin’s 7 Edition Chapter 15. diffuse damage to the alveolar capillary wall B. sinusitis and sinus inversus caused by defect in ciliary motility A. mononuclear cells within the septa C. Neurogenic shock C. Kartagener C. Most common cause of community acquired pneumonia is: A. Cystic fibrosis D. hamartoma th Robbin’s 7 Edition Chapter 17. Page 113 40. fiber waviness Robbin’s 7th Edition Chapter 12. Bronchiectasis 51. Direct extension B. ASD D. 2 weeks Robbin’s 7th Edition Chapter 12. Coronary artery D. Page 584 37 . Chronic mitral stenosis will show this lung finding: A. Pulmonary embolus Robbin’s 7th Edition Chapter 15. Page 589 60. Anemia B. Page 568 59. This congenital heart disease has cyanosis at birth: A. 5-7 days B. Bronchus B. Page 592 C. Deep leg veins th Robbin’s 7 Edition Chapter 4. thickening of the septa B. Valve deformity blood th Robbin’s 7 Edition Chapter 12. Bronchitis Robbin’s 7th Edition Chapter 15. The earliest microscopic sign of myocardial infarction is: A. The physical examination reveals a cardiac murmur. congestion D. What is the clinical impression? A. The usual source of pulmonary embolism is: A.B. Hematogenous route D. Page 579 58. Renal artery C. Page 723 D. Plexogenic arteriopathy B. Medial hypertrophy C. Hemorrhagic infarct is the expected lesion in thromboembolism to: A. The murmur is caused by. Page 138 57. PDA B. Mesenteric artery B. Lymphatic route C. neutrophilic infiltration C. left atrium 53. coagulation necrosis B. Maximal neutrophilic infiltration in a postinfarct heart is: A. 8 hours C. mitral valve D. Page 279 THE HEART For question number 58 – 59 A 25 year old female presents with a history of recurrent fever and arthralgia accompanied by ECG changes and increased ASO titer in the past 2 years. Acute Rheumatic Fever B. neutrophilic infiltrates in septa D. rupture of alveolar septa C. TOF th Robbin’s 7 Edition Chapter 12. Change in consistency of 56. Infective Endocarditis D. It does not metastasize Robbin’s 7th Edition Chapter 7. Aortic Calcific Stenosis C. This finding in the pulmonary vessel would indicate advance pulmonary hypertension A. hyaline membrane coating the septa Robbin’s 7th Edition Chapter 12. Splenic artery Robbin’s 7th Edition Chapter 4. Squamous cell carcinoma would normally spread via A. Internal hyperplasia D. Page 592 55. 1-3 days D. A. VSD C. 54. Contraction deformity D. Page 744 52. Page 136 C. Chronic Rheumatic Heart Disease th Robbin’s 7 Edition Chapter 12. Page 1410 38 . Diagnosis of endocervical polyp D. This breast malignancy has the best prognosis A. Page 336 64. Type I C. Page 339 IMMUNE DYSFUNCTION 66. HCG Robbin’s 7th Edition Chapter 7. A localized area of tissue necrosis resulting from an acute immune complex is an example of this type of hypersensitivity reaction A. Invasive ductal carcinoma Robbin’s 7th Edition Chapter 23. Alpha feto protein B. Klebsiella Robbin’s 7th Edition Chapter 28. B lymphocytes D. Hormonal cytology B. Subarachnoid hemmorhage is associated with this lession A. dendritic cells C. Embryonal carcinoma D. Modified Radical mastectomy B. MRM with frozen section Robbin’s 7th Edition Chapter 23. CA – 125 D. Proliferating blood vessels D. Simple mastectomy C. Influenza D. Identifying etiology of infection C.MALE REPRODUCTIVE SYSTEMS 61. Colloid carcinoma D. Papillary carcinoma B. Degenerative B. Page 210 67. Page 196 68. Choriocarcinoma C. Page 1369 71. anaplastic cells Robbin’s 7th Edition Chapter 28. Natural Killer cells th Robbin’s 7 Edition Chapter 6. Most common testicular tumor in infants A. Tuberculosis B. Excision biopsy D. Interferon B. EXCEPT: A. and CD 8 molecule A. CEA C. Page 210 NERVOUS SYSTEM 69. Type II D. The following are antigen presenting cells. Yolk sac tumor B. Which of the following cells contain CD3. Charcot Bouchard microaneurysm C. If you are suspecting an ovarian cancer. CD4. Page 1139 C. Necrosis C. Type IV th Robbin’s 7 Edition Chapter 6. macrophages D. Intraductal carcinoma 63. This is the most common cause of acute bacterial meningitis in children: A. AV malformation D. Screening of cervical malignancies and its early precursors Robbin’s 7th Edition Chapter 7. rosette formation B. Coli C. H. Histologic lesion that is not present in glioblastoma multiforme A. Page 1366 70. Type III B. Page 1040 FEMALE REPRODUCTIVE SYSTEMS AND BREAST 62. Usefullness of the Paps smear is seen in the following except: A. Berry aneurysm th Robbin’s 7 Edition Chapter 28. you would request for what tumor marker? A. The surgery of choice in a patient with palpable axillary lymph nodes in the presence of proven primary breast carcinoma A. natural killer cells Robbin’s 7th Edition Chapter 6. E. Teratoma Robbin’s 7th Edition Chapter 21. langerhans cells B. Page 1129 65. T lymphocytes C. Page 976 For question number 80 to 83 A 10 year old female presents with generalized edema. Page 735 77. Which among these is considered a premalignant tumor? A. Down the drain Robbin’s 7th Edition Chapter 9. A-V malformation B. Melanoma D. Considering the clinical data.10 (1. Basal cell carcinoma Robbin’s 7th Edition Chapter 25. What is the most likely clinical diagnosis? A. mesangial epithelia D. endothelial cells C. podocytes Robbin’s 7th Edition Chapter 20. Caisson disease is associated with this physical injury A. Which type of mole is noted for its greatest malignant potential? A.015 – 1. X – ray B. Membranoproliferative GN B. Squamous cell carcinoma B. Page 956 39 . This is the major source of oil spill A. Acute glomerulonephritis (GN) C. berry aneurysm C. Up in smoke 78. Compound nevus B. infection Robbin’s 7th Edition Chapter 28. Page 1240 75. Keratoacanthoma C. Acanthosis nigricans th Robbin’s 7 Edition Chapter 25. Page 446 KIDNEY AND URINARY TRACT 79. Chronic renal failure th Robbin’s 7 Edition Chapter 20. Page 1366 SKIN 73.72. Adenocarcinoma C. Off shore drilling D. Electrical injury D. Blast injury C. Acute renal failure B. High altitude sickness Robbin’s 7th Edition Chapter 9. The most common cause of intracerebral hemmorhage is: A. 80. formaldehyde Robbin’s 7th Edition Chapter 15. Melanoma C. asbestos C. Blood pressure is 90/60. Big spills B. mercury D. Seborrheic keratosis D. Page 1240 ENVIRONMENTAL PATHOLOGY 76. Crescent formation is produced by the proliferation of which component of the glomerulus? A. This substance has been implicated in mesothelioma A. Membranous GN D. Nephritic syndrome D. Intradermal nevus D. Which among these is not a common malignancy of the skin? A. Actinic keratosis B. Page 1234 74. Junctional nevus Robbin’s 7th Edition Chapter 25.022) Protein = 3+ Oval fat bodies = 3+ 81. Page 978 C. Normal glumerulous Robbin’s 7th Edition Chapter 20. microaneurysm D. parietal epithelia B. what is the most likely light microscopic pattern? A. Decompression disease B. Nephrotic syndrome Urinalysis showed the following findings: specific gravity = 1. Page 431 C. 82. Linear C. The clinical symptoms of this patient are referable to which of the following mechanisms? A. Negative immunoflourescene B. Urethral fibrosis & stricture B. Subepithelial deposits C. Benign prostatic hyperplasia C. No pathologic EM findings Robbin’s 7th Edition Chapter 20. Anemia due to hemoglobinopathy D. What is the expected electron microscopic (EM) findings? A. A. Subendothelial deposits D. Increased urine output C. The red cell morphology of the peripheral smear is expected to be A. Page 624 88. The most likely RBC disorder present in this patient would be: A. The mechanism for the abnormal RBC parameter is due to. Iron deficiency anemia B. Normocytic normochromic C. Cystitis D. Anemia due to chronic blood loss B. Digital rectal examination revealed an enlarged prostate gland. Granular D. Page 969 83. Microcytic hypochromic Robbin’s 7th Edition Chapter 13. RBC destruction due to antibodies B. An elevated serum prostatic specific antigen (PSA) together with an elevated alkaline phosphatase level is consistent with which of the following mechanisms? A. Prostatitis Robbin’s 7th Edition Chapter 21. Thalassemia D. Folic Acid Deficiency C. 84. Prostatic irritation and inflammation D. Megaloblastic anemia C. Page 642 MUSCULOSKELETAL SYSTEM 40 . Page 643 89. Macrocytic hyperchromic B. Page 1033 NUTRITIONAL DISORDERS For question number 86 to 93 A 35 year old female came in for consultation due to increased amount and frequent menstruation for the past 6 months. Lack of dietary iron D. 86. CBC was done revealing low hemoglobin. Anemia due to decreased RBC production Robbin’s 7th Edition Chapter 13. Prostatic malignancy B. Variably linear granular Robbin’s 7th Edition Chapter 20. Sickle Cell Disease th Robbin’s 7 Edition Chapter 13. Anemia due to hemolysis C. Page 624 87. What is the expected immunofluorescence pattern in this case? A. Urinary bladder outlet obstruction Robbin’s 7th Edition Chapter 21. hypochromic type of anemia is seen in which of the following conditions? A. Normocytic hypochronic D. Macrocytic. Loss of podocyte foot process B. Page 970 For question number 84 to 85 A 65 years old man sought consult due to decreased vigor urinary stream and increased urinary frequency. RBC membrane defect th Robbin’s 7 Edition Chapter 13. hematocrit and red blood cell (RBC) count. Page 1053 85. lytic punched – out lesion are seen. direct cytopathic effect B. Most common primary malignant tumor of the bone A. hemosiderin D. Chondrosarcoma B. Osteosarcoma C. This tumor is probably made up of A. neither A nor B is correct 3. This soft tissue tumor has biphasic feature of tumor cells A. 41 Inflammation and Repair 5. The sequence of cellular events in inflammation is: A. Giant cell tumor Robbin’s 7th Edition Chapter 26. immune – mediated reactions Robbins 7th edition p. Cellular Injury and Adaptation 1. melanin Robbins 7th edition p. On x-ray of the skull. denaturation of proteins B. plasma cells th Robbin’s 7 Edition Chapter 14. Fibrosarcoma C. 356 – 357 Process (es) involved in cell necrosis is (are): A. The following statements are TRUE of caseous necrosis: A. Osteosarcoma C. chemotaxis  margination  diapedesis  phagocytosis D.90. The most effective means of microbial killing is via the: A. bilirubin C. Osteosarcoma B. Metastatic tumor D. Page 1323 CHOOSE THE BEST ANSWER. Synovial sarcoma D. This lesion has the oldest age group of predilection A. proliferating chondrocytes D. giant cells C. both A & B are correct D. chemotaxis  diapedesis  margination  phagocytosis th Robbins 7 edition p. hydrogen peroxide. C. Viruses induce cell injury through: A. Giant cell tumor th Robbin’s 7 Edition Chapter 26. neither A nor B is correct th Robbins 7 edition pp 21 -22 4. 21 – 22 C. margination  diapedesis  chemotaxis  phagocytosis B. enzymic digestion of cells th Robbins 7 edition pp. Page 679 92. Ewings sarcoma D. Jaundice is due to the accumulation of this pigment: A. Malignant fibrous histiocytoma Robbin’s 7th Edition Chapter 26. only A & C are correct th Robbins 7 edition 59 . both A & B are correct D. bone forming tumor cells B. it is a combination of coagulation and liquefaction necrosis B. lipofuscin B. 53 6. Page 1298 93. neither A nor B is correct 2. diapedesis  margination  chemotaxis  phagocytosis C. myeloperoxidase and halide system D.61 41 . both A & B are correct D. Page 1294 91. oxygen – independent mechanism B. myeloperozidase independed mechanism C. Chondrosarcoma B. it is a characteristic feature of tuberculosis C. two B. exudate C. presence of foreign bodies D.120 Genetics Disorders 12. Fluid that collects during acute inflammation and that has a protein content in excess of 3. 174 . non-disjunction in meiotic division in spermatogenesis C. Periorbital edema is usually encountered in: A. Ascaris lumbricoides D. hydropericardium B. inversion B. The following are suppressor genes EXCEPT: A. p53 C. NF – I th Robbins 7 edition p. translocation Robbins 7th edition p. Keloid is due to excessive: A. decreased oncotic pressure of plasma protein D. The most important cause of delay in wound healing is: A. A 6-year-old boy develops abdominal pain and vomiting preceded by a 3-day period without bowel movements. mutation C. 114 8. none C. three th Robbins 7 edition p. translocation D. increased osmotic pressure of interstitial fluid (sodium retention) C. inflammatory states th Robbins 7 edition p. amyloid deposition D. filariasis C.175 The most common cause of the occurrence of Klinefelter’s syndrome: A. A segment of one chromosome transferred to another segment is known as: A. increased hydrostatic pressure of the blood B.178 15. inadequate blood supply B.300 Infectious Disease 16. 14. deletion Robbins 7th edition pp. transudate D. liver cirrhosis B. 179 13. non-disjunction in meiotic division in embryogenesis D. Strongyloides tercoralis B. bcl-2 D. Taenia saginata C. one D. granuloma formation Robbins 7th edition 115 C. nephritic syndrome D.015 is termed: A.7. wheal Robbins 7th edition p. granulation tissue B. infection C.0 g/L and SG over 1. Edema can result from any of the following mechanisms: A. Onchocerca volvulus th Robbins 7 edition p. the most likely reason for the obstruction is a bolus of: A. all of the above are correct Robbins 7th edition p. Of the following. A diagnosis of intestinal obstruction is made. The number of Barr bodies in a patient with 48 XXXY karyotype: A. collagen accumulation Fluid and Hemodynamics 9. mechanical factors Robbins 7th edition p. 298 .352 42 . non-disjunction in meiotic division in oogenesis B. 351 . 120 10. 122 11. APC B. 3rd trimester of pregnancy B. 452 . Vitamin E th Robbins 7 edition p. Function of thiamine: A. A. the ileum th Robbins 7 edition p. The mechanism of clotting is affected by: A. cheery-red discoloration of skin Robbins 7th edition p. the gall bladder B. myocardial infarction C.453 20. 2nd trimester of pregnancy B. 1st trimester of pregnancy C. respiratory enzyme in the cytochrome enzyme D. peptic ulcer disease Robbins 7th edition p. all of the above are correct Robbins 7th edition pp. carbon monoxide B. all of the above th Robbins 7 edition p. 432 22. fatty liver D. 456 Environmental Pathology 21. chloroform D. progressive massive fibrosis B. 835 Nutritional Disease 18. all of the above are correct th Robbins 7 edition p. the cortex of the kidney D. 421 C. parathyroid gland D. 470 26. 2nd trimester of pregnancy D. the appendix C. the end of 1st trimester D. Thiamine B. 419 24. Ethyl alcohol toxicity will cause: A. Vitamin A C. injuries resulting from accidents B. co-enzyme in oxidative decarboxylation of alpha-ketoacids B. the middle of 1st trimester 43 . coal worker’s pneumoconiosis D. Exposure to carbon particle in the ambient air will result to: A. congenital anomalies D. Which of the following is the most common adverse effect of smoking? A. synthesis of DNA and RNA C. cardiac diseases Robbins 7th edition p. gingivitis 23. The leading cause of death in children between 5-14 years of age is: A. vitamin D C. The metabolism of calcium is closely regulated by: A.473 27. 732 . Anthracosis C. Organogenesis is completed by: A. 456 19. thyroid gland B. resistant to natural process of decay.17. the end of 2nd trimester C. cancer of the bladder D. Vitamin K D. venous thrombosis B. This/these pollutants is/are dangerous because once released into the environment they are “forever” i. malignant neoplasm C. The highest at risk period for intrauterine cytomegalovirus infection: A. cancer of the pancreas B. polychlorinated biphenyls C.734 Diseases of Infancy & Childhood 25.e. all of the above are correct Robbins 7th edition p. A chronic carrier state of typhoid fever is most likely due to the persistence of the organism in: A. any of the above th Robbins 7 edition pp. anthracosis D. Lymph nodes and Spleen 31. Asbestosis C. “Starry sky” pattern of lymph node is characteristic of: A. 256 . immunologic disorders Robbins 7th edition p. Kaposi’s sarcoma D. usually male C. 717 44 . 701 . unifocal langerhans cell histiocytosis th Robbins 7 edition p. 686 & 688 32. Lymphaniosarcoma C. lymphocytic predominance C. Hemangiopericytoma th Robbins 7 edition pp. Histiocytic lymphoma B. drug abuse B.678 Lacunar cells are characteristics of this type of Hodgkin’s disease: A. chronic bronchitis C. Alpha-1 antitrypsin deficiency is associated with: A. A biopsy of the femoral artery revealed calcification in the media would be compatible with: A. cancers D. Sinus histiocytosis are normally seen in nodes draining: A.746 Respiratory System 36. 470 .702 34. 666 35. Complicated or advanced atherosclerosis would most likely affect this portion of the blood vessel: A. intima and adventitia B.Robbins 7th edition pp. eosinophilic granuloma C. 515 30. panlobular emphysema C. accelerated hypertension B. A diagnosis worth considering in this clinical picture is: A. Wegener’s granulomatosis B. Sezary’s syndrome Robbins 7th edition p. none of the above th Robbins 7 edition p. 677 . Good pasture’s syndrome B. emphysema D.520. adventitia th Robbins 7 edition p 517 . Enlargement of pulmonary alveolar spaces with destruction of septal walls is seen in: A. intima only C. 33. intima and media D. 745 .471 The Heart 28. young individuals. Angiosarcoma B. usual complication of ischemia and gangrene D. alveolar proteinosis Robbins 7th edition p. lymphocyte depletion D. bronchogenic carcinoma th Robbins 7 edition pp. tuberculosis th Robbins 7 edition pp.257 WBC. nodular sclerosis th Robbins 7 edition p. A vascular tumor associated with AIDS is: A. Hodgkins lymphoma D.719 37. primary atypical pneumonia D. 718 . A patient presents with hemoptysis and acute renal failure. The most common type of Histiocytosis X in infants is: A. 29. letterer-siwe syndrome B. pulmonary infarction B. infection C. hand-schuller-christian disease D. mixed cellularity B. Burkitt’s lymphoma C. 806 . The least common type of gall bladder calculi are: A. arises from the pancreatic ductal cells D. pure calcium carbonate D. Hydrops of the gall bladder B. bronchogenic carcinoma C. Small intestine th Robbins 7 edition p. 933 45. Elevated serum gastrin levels D.1034 47.38. Pylorus B. Whipple’s disease B. Biliary & Pancreas 43.285 Liver. 844 41. 929 . histoplasmosis Robbins 7th edition p. pure calcium bilirubinate C. Which of the following condition(s) increase(s) the risk of bile duct carcinoma? A. Choledochal cyst C.951 46. mixed stone Robbins 7th edition p. Which statement about esophageal cancer is false? A. Choledochal cyst C. Choledocholithiasis th Robbins 7 edition p. most common in the distal portion C.809 40. Macrophages with positive periodic acid-schiff-staining material in intestinal lamina propia and lymph nodes are characteristically found in: A. Cholelithiasis is mainly due to: A. is associated with hypocalcemia C. Infection B. pure cholesterol B. abscess B. is usually cured by total pancreatectomy th Robbins 7 edition p. Stomach D. Porcelain gall bladder D. Gardener’s syndrome Robbins 7th edition p. Carcinoma of the pancreas: A. inflammation 45 . Crohn’s disease B. villous adenoma of the colon D. Wilson’s disease C. Carcinoma of the gall bladder th Robbins 7 edition p. supersaturation of bile with cholesterol Robbins 7th edition p. familial polyposis of the colon C. primarily a disease of elderly males D. A chest x-ray that shows a shaggy cavity with a thick irregular border and satellite densities in the right lower lobe is most compatible with: A. occurs most often in the body of the pancreas B. 950 . The Mallory-Weiss syndrome is due to laceration in the mucosa of the: A. Hydrops of the gall bladder B. Carcinoma of the GIT would be expected to occur with greatest frequency in patients with: A. 933 44. 284 . 753 GIT 39. 1033 . bile stasis D. characterized by dysphagia and painless weight loss th Robbins 7 edition p. Chronic obstruction of the cystic duct results to development of: A. Porcelain gall bladder D. Esophagus C. it is usually of squamous cell type B. 802 42.930 C. gastric ulcer Robbins 7th edition p. tuberculosis D. Giant condyloma C. one does not belong: A. These nests of urothelium are called: A. male sex C. The majority of ovarian neoplasms arise from: A. congenital abnormalities of the urinary tract D. severe hypocomplementemia B. 1026 54. renal papillary necrosis D. This is most probably due to: A. the other main cause of acute tubular necrosis is: A. H. 526 51. recto-vaginal septum B. coarctation of the aorta and acromegaly are all conditions that may lead to: A. Ischemia D.1084 57. 1083 . acute episodes of paroxysmal HPN B. ectopic pregnancy D. Brunn’s nests B. pyonephrosis th Robbins 7 edition p. hypersensitivity vasculitis D. mole Robbins 7th edition p. 998 . The most common site of endometriosis: A. metastatic from GIT th Robbins 7 edition p. abortion C. pregnancy th Robbins 7 edition pp. Choriocarcinoma is most often preceded by: A. 998 . Of the complications of acute pyelonephritis enumerated below. Brunner’s nests C. Sepsis C. A newborn baby was noted to pass out urine through a small opening in the area of the umbilicus. hypertension th Robbins 7 edition p. Exstrophy of the bladder B. surface epithelial cell D. Lower Urinary Tract & Male Genital System 52. Burner’s nests Robbins 7th edition p.1000 Chronic renal disease. 1110 46 . ovary D. The following is caused by human papilloma virus type 6 (HPV-6): A. Wegener’s granulomatosis B. Patent urachus D. In addition to nephritic injury. 933 49. Condyloma acuminatum D.Kidney 48. 1024 53. germ cells C. Of the risk factors for pyelonephritis enumerated below. pheochromocytoma. Nests of urothelium may be found in the lamina propia of the urinary bladder representing normal variation in the morphology of the bladder. all of the above Robbins 7th edition p. retroperitoneal abscess formation C. normal pregnancy B. fallopian tube C. Urachal cyst C. sex-cord stroma B. condyloma lata B. vagina Robbins 7th edition p. diabetes mellitus B. 1265 Female Genital Tract 55. Conn’s syndrome.1000 50. Any of the above th Robbins 7 edition p. 1093 56. venous thrombosis C. one does not belong: A. Burney’s nests D. immune complex deposition Robbins 7th edition p. 1083 . suppression of cortisol secretion C. upper inner quadrant D. no suppression of cortisol secretion B. pustule D. mucinous cystadenocarcinoma D. The most common primary malignant tumor of the ovary: A.1209 Skin & Musculoskeletal 66.1084 Breast 59.1209 64. 1139 60. upper outer quadrant B. axillary lymph nodes C.58. brain th Robbins 7 edition p. 65. one would expect: A. 1162 63. 1229 67. Plasma cortisol was elevated. pustules th Robbins 7 edition p. wheal C. If this were Cushing’s syndrome. only A & B are correct Robbins 7th edition p. Cushing’s syndrome D. Which of the following is a secondary lesion? A. 1162. pituitary ACTH excess B. both A & B are correct D. Only B & C are correct th Robbins 7 edition p. 1150 61. mucinous carcinoma C. papules B. 1207 . serous cystadenocarcinoma B. ectopic ACTH syndrome D.1207 . Psoriatic patients typically develop silvery: A. Waterhouse-Friderichsen syndrome B. scales D. 1209 If you suspect the presence of Cushing’s syndrome. suppression of ACTH secretion D.” truncal obesity and hisrutism. 1123 Endocrine For numbers 87 to 89 62. all of the above are correct th Robbins 7 edition p. Suppression of cortisol secretion with high dose dexamethasone suppression test indicates the presence of: A. neither A nor B is correct th Robbins 7 edition p. liver B. The most frequent site of breast cancer is: A. These features characterize: A. high dose dexamethasone suppression test C. subareolar Robbins 7th edition p. lower inner quadrant C. paget’s disease of the breast th Robbins 7 edition p. you should perform a: A. p. 1256 47 . an adrenal neoplasm C. Cord like ducts filled with necrotic and cheesy tumorous tissue that can be readily extruded upon pressure is seen in: A. erosion B. bulla th Robbins 7 edition p. Conn’s syndrome C. lungs and bone D. excoriations C. A female patient presented with “moon facies. Sites of metastases of cystosarcoma phyllodes are usually: A. low dose dexamethasone test B. yolk sac tumor th Robbins 7 edition p. comedocarcinoma D. medullary carcinoma B. endometrioid carcinoma C. 71.68. porphyria Robbins 7th edition p. gram stain C. Pityrosporum C. One of the etiologic factors in acne is the: A. Propionibacterium B. at what point of the cell cycle is blocked? 48 . A simple lab procedure to do to support a diagnosis of herpes zoster is: A. KOH stain D. 387 . Enterococci th Robbins 7 edition p. Tzanck smear B. papules. Patch test th Robbins 7 edition p. anaplasia C. Dapsone 100mg daily for 6 months. Supratentorial C. In hypertrophy. reddish. ependymoma Robbins 7th edition p. vesicles. Medulloblastoma C. Atrophy B. Posterior fossa B. glioblastoma multiforme D. Histologic criteria for glioblastoma multiforme: A. 1407 75. and bullae. 1255 . Medulloblastoma most frequently occurs in: A. Metaplasia _____ 2. The patient most likely has: A. vascular endothelial proliferation B. Most common primary malignant brain tumor: A. 1401 . psoriasis D. 1372 CHOOSE THE BEST ANSWER: _____ 1. middle adulthood D. Rifampicin 600mg once a month for 6 months. Which cell adaptation is achieved by decreasing cellular activity and reducing number and size of cellular organelles? A. Staphylococcus D. Hyperplasia C. lichen planus B. Clofazimine 50mg daily for 6 months th Robbins 7 edition p. Tuberculoma in children frequently occurs in: A. Ofloxacin 400 mg. erythema multiforme C. both A & B are correct D.1402 74. A patient presents with vesicles on the left side of his trunk.388 70. Ganglioneuroma B. Rifampicin 600mg once a month for 6 months. with pale eroded center involving the extremities. Minocycline 100mg daily for 6 months B. A child presents with an array of macules. children & young adults B.1256 Nervous System 72. age 60 and above C. 368 69. neither A nor B is correct Robbins 7th edition p. Hypertrophy D. There was intake of penicillin and sulfonamides for urinary tract infection. Clofazimine 50mg daily for 2 years D. Dapsone 100mg daily for 6 months C. Infratentorial D. no age predilection is observed Robbins 7th edition p. Transtentorial th Robbins 7 edition p. 1407 73. lips and oral mucosa in a symmetric pattern. Dapsone 100mg daily for 2 years. Rifampicin 600mg once a month for 2 years. 1264 The paucibacillary therapy for Hansen’s disease is: A. Rifampicin 600 mg. Caseous B. Autolysis C. Anaerobic respiration C. Chloride C. Ubiquitin _____ 9. Caspases B. Which of the following is a definite sign of malignancy? A.A. Dystrophic calcification D. In the TNM staging system. M stands for which of the following? A. Transplantation of organs between two identical twins is called: A. Autophagy of organelles D. Granulosum C. Excessive proliferation of cells D. Apoptotic calcification C. Allograft B. Abscess of the liver is a form of which type of necrosis? A. Gene amplification C. Apoptosis B. Autograft C. Calcium B. In hyperplasia of the skin. Synthesis of growth factor receptors 49 . Cell membrane C. Morphologically. Infiltrative growth D. Gangrenous calcification B. a malignant tumor is poorly differentiated when there is: A. Chaperones C. Potassium D. M to G0 _____ 3. Rapid growth _____ 11. Xenograft ____ 10. Sodium _____ 6. Which of the following increases the cell’s ability to adapt and survive prolonged environmental changes? A. Ras oncogene qualitatively changes function of proto-oncogene through which form of mutation? A. Laminin D. The deposition of calcium salts and other mineral salts in vital tissue that reflects derangement in calcium metabolism and hypercalcemia is called: A. Metastasis to distant sites C. Which facilitates the degradation of denatured protein that is beyond repair to prevent further injury to the cell? A. Compression of adjacent structures C. Poor resemblance to cell of origin D. Which of the following is an essential component of neoplasia? A. Very high nucleo-cytoplasmic ratio _____ 13. Gangrenous Liquefactive D. Entering the cell cycle D. Mitochondrial DNA B. Regional lymph node metastasis _____ 16. S to G0 B. Subdivision and multiplication B. Metastasis B. Accumulation of lactic acid due to anoxia stops the synthesis of proteins by disrupting which cellular structure? A. Translocation _____ 15. Basale B. Lucidum D. G0 to G1 D. Endoplasmic reticulum D. Nuclear DNA _____ 7. which stratum enters the cell cycle? A. Extrusion of organelles _____ 4. Point mutation D. Probable morbidity D. Metastatic calcification _____ 12. Total lost of normal function of cells B. _____ 8. Autonomic growth C. Isograft D. Markedly hyperchromatic nuclei C. Large mass B. Spinosum _____5. Probable mortality B. Which of the following substances disrupt the steady state and causes cell swelling? A. Vascular invasion _____ 14. Deletion B. Coagulative C. Which mechanism explains the initial reduction in size of an organ undergoing atrophy? A. S C. Acquiring faster amoeboid movement to run after the offending agent B. Loss of function of increasing number of fully differentiated cells D. Synthesis of basement membrane _____ 19. Acquired immune response B. the antiviral activity of interferon reduces the synthesis of: A. Host proteins in the ribosomes B. The pathogenesis of coagulation necrosis is: A. Removal of cellular debris B. Large areas of necrosis and exudate formation D. scar formation is primarily due to the presence of: A. Regeneration of parenchymal cells D. The formation of giant cells in a granulomatous inflammation is for the purpose of: A. Host and viral mRNA D. Cells that lost ability to enter the G0 phase of cell cycle and are perpetually in the cycle B. Antigen that is not readily digested B. Action of powerful proteolytic enzymes from lysosomes and inflammatory cells B. Host and viral DNA C. Forming a barrier around the offending agent D. Increasing ligands for Fas or death receptor C. Memory cells are produced _____ 18. Main cellular components are the T-lymphocytes C. The repair component of healing involves principally which of the following? A. Which of the following best describes dysplasia in the context of biological growth? A. Presenting more efficiently the antigen-MHC complex _____ 21. Which statement best expresses the concept of chronic inflammation? A. Production of oxygen-derived free radicals _____ 25. Oxygen deprivation _____ 20. Rapid entry of water and calcium with saponification of cellular contents _____ 26. Loss of differentiation and/or maturation of increasing number of cells C. Aging causes cell damage and death through which mechanism? A._____ 17. The ultimate goal of homeostasis in a persistently changing interstitial milieu is to: A. Main humoral components are the interferons D. Macrophages play a dominant role B. Which is the unique characteristic of innate immune response? A. In innate immune response. Result of a failed acute inflammatory response _____ 22. Formation of supporting connective tissue C. Inactivation of autolytic or intracellular enzymes and dehydration without cell decomposition D. Limit entry and exit of substances into the cytosol C. Viral cell membrane proteins _____ 23. Mutation of nuclear DNA D. Acquiring greater capability for phagocytosis C. In acute inflammation. Activation of the Fas or death receptor C. Malignant cells that already possess invasive capabilities but are still in the resting phase of the cell cycle 50 . Maintain constancy in the cytosol to ensure physiologic function D. Infection C. Remove cells that cannot adapt ______ 24. Induce cell division B. Persistence of stimulus with prolonged inflammation and repair D. Accumulation of intracellular calcium B. Inflammatory response following acute inflammation C. B. Destruction of follicles by Tc lymphocytes and subsequent excessive release of thyroid hormones D. Excessive production of IgG thyroid-stimulating antibodies due to defect in the Ts lymphocytes _____ 28. DNA fragmentation by endonuclease activation D. Which of the following makes homeostasis possible in the body systems? A. Deactivate the cyclin-dependent kinase complexes thus shortening the cell cycle so that more cells are produced within a shorter time B. Constriction due to edema of the interstitial tissue C. Deactivae the p53 suppressor gene so tat no delay in DNA synthesis occurs C._____ 27. Numerous fractures and breakups of membranes of cell and saccular organelles C. Internal environment always constant B. The key mechanism of apoptosis is: A. Cellular swelling of the lining epithelium with paralysis of the cilia B. Grave’s disease is cause by which of the following immunologic reactions? A. Tight coiling of nuclear DNA threads _____ 34. _____32. Destruction of follicles by autoantibodies against thyroglobulin C. In an injured liver. TNFα) released by inflammatory and tumor cells are believed to cause cancer cachexia through which of the following mechanisms? A. Polyribosome complexes dispersed into single ribosome D. A Type I hypersensitivity reaction due to the intake of an antibiotic can immediately kill a person because of: A. Spasm of the smooth muscles and increased mucus gland secretion _____ 33. Infectious agent triggers premature release of T-lymphocytes from the thymus still possessing receptors to self-antigens C. Tendency of body system to be in steady state _____ 31. Non-specific degradation & random DNA cleavage _____ 30. D. Which is the mechanism by which an infectious agent may trigger an autoimmune disease? A. Necrosis and desquamation of the lining epithelium D. Increased interstitial osmotic pressure B. Increased hydrostatic pressure D. Decreased plasma oncotic pressure C. Enter and complete the cell cycle D. Marked swelling of mitochondria and clumping of mitochondrial DNA B. Soluble factor (e. Tendency of body system to be in equilibrium D. Infectious agent has a similar antigenic structure as self-antigen and the resulting immune response cross-reacts with self-antigen. Which is the initial step in transudate formation? A. Infectious agent triggers macrophages to randomly present antigens to the Tlymphocytes and by chance present an antigen that is similar to self-antigen. Cessation of mRNA synthesis B. growth factors ultimately cause liver cells to: A. Increased vascular permeability _____ 29. The point of irreversibility of cell damage is evident ultrastructurally by the presence of which of the following? A. Activation of natural killer cells due to compliment fixation on the follicular cells B. Increase tumor needs for essential nutrients at the expense of normal cells 51 . Divert TP of normal cells to meet increased energy needs of tumor cells B. Coiling of DNA chromatin C. Physiologic feedback mechanism C.g. Grow in sized by increasing the cytoplasmic organelles _____ 35. Infectious agent suppresses the suppressor T-lymphocytes thus giving the T-helper lymphocyte the “freedom” to react to any antigen including self-antigen. The basal cell changes seen in reflux esophagitis is an example of this adaptive change: A. Hypertrophy D. Memory T-lymphocytes continue to recruit uncommitted T-lymphocytes even in the absence of the antigen D. Th lymphocytes do not have the receptors for this virus. Lymphatic flow resulting in reduced interstitial fluid B. Suppurative inflammation 52 . and so. The difficulty in producing an effective vaccine against HIV is due to which of the following? A. Erosions C. Patients on anti-cancer chemotherapy are highly susceptible to infections because of: A. Metaplasia _____ 42. Thrombosis D. Congestion and edema B. In an acquired immune response to a specific antigen. After initial exposure. The frequent enzymatic errors in transcription. Marked edema formation resulting in the compression of arterioles C. which make copies if the virus’ RNA. Non-distended segment proximal to the distension _____ 44. Recruitment of uncommitted T-lymphocytes is faster in the second exposure _____ 38. Passive congestion C. Interfere with normal cell metabolism and metabolism of essential nutrients D. Proteinaceous debris from dead bacteria _____ 37. Hemorrhage D.C. Suppress desire for food by acting on the pituitary _____ 36. Susceptibility of liver to drug injury resulting in diminished complement production C. all T-lymphocytes develop receptors tot he antigen and are ready to react to a second exposure B. Atrophy B. Toxicity of the drugs causes the neutrophils to lose its phagocytic activity D. Distended portion C. The lesion of cholera is at most which of the following: A. the interstitial osmotic pressure progressively increases due to the increasing amount of: A. results in different stains B. Non-distended distal segment D. Marked diminution in the number of cells of the innate and acquired immune response B. Neutrophils C. In the vascular phase of acute inflammation. Growth and multiplication of the organism in the blood resulting in obstruction to blood flow B. In megacolon. Suppuration resulting in the destruction of blood vessels _____ 41. Transverse colon B. Toxicity of the drugs impairs the production of MHC by macrophages _____ 40. Hyperplasia C. For some unknown reasons. Memory T lymphocytes committed to that antigen are already present C. Production of large amount of toxins and enzymes by the microorganism resulting in denaturation of intracellular proteins D. the reaction to a second exposure usually takes a much shorter time compared to the first exposure because of which of the following? A. It is difficult to isolate the antigen of an RNA virus _____ 39. The exact structure of the HIV has not been determined yet E. Embolism _____ 43. The formation of esophageal varices secondary to cirrhosis is primarily due to: A. cannot be activated to initiate an immure response F. Coagulation necrosis can be seen in some types of bacterial and fungal infection when there is: A. Active congestion B. Plasma proteins D. the aganglionic segment is in which portion: A. Localized deficiency of Protein C and S _____ 53. A right-to-left shunt best exemplified by which of the following? A. Luminal obstruction _____ 52. Age of patient B. Plasma cells _____ 47. The inflammatory reaction seen in typhoid ileitis is composed mostly of: A. Ascending C. Accumulation of clotting factors C. Disorganized benign glands C. Dysplastic glandular epithelium D. Ischemia B. The prognosis of colonic carcinoma is more dependent on: A. Basal cell carcinoma is characterized by which of the following? A. A 10 month old infant presented with a vaginal mass. Rhabdomyosarcoma B. Chronic C. Necrotizing D. Metastatic adneocarcinoma B. Hamartomatous growth B. Chondrosarcoma C. Locally invasive and rarely metastasizing tumor D. All soft tissue tumors are positive for: A. What is the most likely diagnosis? A. Larger amount of lymphoid tissue organized into nodules _____ 46. Subacute _____ 54. Presence of intercellular bridges and keratohyalin _____ 56. Deficiency of plasmin D. Descending D. It is the longest segment D. The inflammatory lesion seen in acute appendicitis is usually initiated by: A. Colonic carcinoma is most common at this portion: A. Recto-sigmoid _____ 50. Trophozoites penetrate the mucosa but do not penetrate the muscularis propria D. Intestinal contents is less liquid and alkaline. Mononuclear cells C. In infective endocarditis. TOF D. Ewing’s sarcoma D. Early metastasis hematogenous spread C. ASD B. Vimentin 53 . Cecum B. PDA C. Eosinophils B. S-100 D. Fecal material within the lumen D. VSD _____ 55. The essence of adenomas of the gastrointestinal tract is: A. Desmin C. Aggressive tumor B. Neutrophils D. Upper third of the mucosa adjacent to the ulcer regenerates above the ulcer _____ 48. Bacterial infection C. Ischemia causes the lower third of the mucosa to die first C. The most common primary cancer of the bone is: A. The ulcers of amebiasis are initially flask-shaped because: A. The reason the typhoid lesion is usually seen in the ileum is that: A. Location D. Acute B. Absorption is greatest in this segment B._____ 45. Synovial carcinoma _____ 58. Liposarcoma D. Grade of tumor C. Polypoid mucosal outgrowth _____ 49. Fibrosarcoma C. organisms of low virulence that cause infection in a previously abnormal. Osteosarcoma _____ 57. deformed valves is classified as: A. Endothelial injury B. Cytokeratin B. thus favoring bacterial growth C. Stage _____ 51. The thrombosis seen in the veins of hemorrhoids is primarily due to this mechanism: A. Fibrosis causes the upper portion of the mucosa adjacent to the ulcer to slowly approximate each other over the ulcer B. dorsal “buffalo hump” and thin extremities suggests: A. Mucosa and submucosa D. Adenocarcinoma C. Proteus vulgaris _____ 64. Liver B. pyogenes. Typhoid ileitis _____ 67. Spleen D. Mucosa. dysuria. Cri du chat syndrome B. He was moaning. Minimal change disease D. and holding his abdomen. E. “moon face”. and inner smooth muscle layer B. Addison’s disease C. N. Pancreatic lipases 54 . The likelihood that a clinical condition will occur when a particular genotype is present: A. Conn’s syndrome D. Familial adenomatous polyposis C. writhing in pain. Which of the following is an autosomal deletion disorder? A. submucosa. coli B. Which is a characteristic finding in alcoholic liver disease? A. Gastrointestinal stromal tumor D._____ 59. Throat culture revealed S. gonorrhea D. Nephritic syndrome B. H. HbsAg B. Neutrophils _____ 66. If autopsy is done. Poststreptococcal glomerulomephritis B. Down's syndrome C. Patau's syndrome _____71. Stomach _____ 61. Rapidly progressive glomerulonephritis _____ 63. Digestive proteases D. Sipple’s syndrome _____ 62. Ground glass hepatocytes D. Bacterial toxins C. All layers including the outer smooth muscle layer _____ 69. Peutz-Jeghers syndrome B. Penetrance D. Adenomatous polyp B. Polymorphism ____ 72. Nephrotic syndrome _____ 65. atrophic tubules with eosinophilic casts all suggest: A. Membranous glomerulonephritis C. urinary frequency and flank tenderness. Heterozygosity B. Acute pyelonephritis C. Coarse asymmetric renal corticomedullary scarring. A 19 yr. Chronic pyelonephritis D. Which of the following is likely to be elevated? A. Eosinophils C. Edward's syndrome D.old female student developed acute pharyngitis. Mosaicism C. deformity of the renal pelvis and calyces. Ulcerative colitis _____ 68. Urinalysis showed abundant WBC and bacteria. was brought to the ER. Crohn’s disease D. Alpha feto protein C. Mallory bodies _____ 60. influenzae C. Pancreas C. The most common type of carcinoma associated with Barrett’s esophagus is: A. Crescents formation seen in renal biopsy strongly suggests: A. Cholera C. Cushing’s syndrome B. A 28 y/o woman preented with fever. A 24 yr old male. hours after excessive food and alcohol intake. Basophils B. Mucosa C. In liquefactive necrosis. Intestinal intussusception in older children and in adults is most likely associated with: A. The most likely etiology is: A. which organ will most likely show necrosis? A. Inflammatory cells B. destruction of the tissue morphology is mainly due to: A. Serum amylase is elevated. Redistribution of body fat. Lymphocytes D. Leiomyosarcoma B. Which of the following has highest risk of developing colorectal carcinoma? A. Squamous carcinoma _____ 70. The erosions associated with acute gastritis is seen in this layer/s of the stomach: A. Hirschprung’s disease D. Brain B. Hyperplasia C. Infected with Mycobacterium tuberculosis C. Increased hydrostatic pressure D. Liquefactive necrosis ____ 85. Cause extensive bleeding C. Which of the following is a reflection of left heart failure? A. Lost innervation ____ 75. In females. high grade fever 55 . Na+ accumulation B. dyspnea B. Infected with Pseudomonas aeruginosa D. Splenomegaly ____ 84. Multifocal lesions D. Hypovolemic C. Anesthetic accident resulting in loss of vascular tone and peripheral pooling of blood leads to which type of shock: A. Severe combined immunodeficiency B. most cancer deaths are caused by malignancy in the : A. Most likely condition that a histologically benign neoplasms may be fatal: A. Increased hydrostatic pressure D. Pitting edema D. Contralateral breast C. In which condition will caseation necrosis most likely occur? A. Metaplasia ____ 76. Ipsilateral axillary lymph node D. the most common form of immunodeficiency is: A. Septic ____ 81. Apoptosis B. ___ 74. Decreased oncotic pressure C. Hepatomegaly C.____ 73. Dysplasia B. Chronic granulomatous disease D. Uterus ____ 78. In acute congestive heart failure. Which of the following morphologic changes is irreversible? A. Decreased plasma oncotic pressure C. Dysplasia C. Increased vascular permeability ____ 82. Which is the initial mechanism of transudate formation? A. Degradation of DNA to nucleosome-sized fragments is characterisitc of: A. Metaplasia Neoplasia D. Lung D. edema of the lower extremities is due to: A. Fail to invoke immune response B. The erosion of progressive pulmonary tuberculous cavitary lesions into nearby pulmonary blood vessels would present clinically as: A. Colorectal segment C. Cardiogenic B. Coagulative necrosis B. Ischemic injury to the myocardium lasting for 15 minutes leads to: A. Transform into cancer ____ 79. hematemesis C. X-linked agammaglobulinemia ____ 80. Liquefactive necrosis D. Pulmonary edema B. Increased interstitial osmotic pressure B. Ataxia-Telangiectasia C. hemoptysis D. Arterial blood supply interrupted by thrombus B. Fat necrosis C. Neurogenic D. In the Philippines. Lungs ____ 77. Breast B. Pump failure ____ 83. Invasive ductal carcinoma in the upper outer quadrant of the right breast will likely metastasize to: A. Gangrene D. Previous trauma D. Colonic adenocarcinoma: Carcinoembryonic antigen B. Lymphatics D. Increased hydrostatic pressure ____ 87. the most common source of infection with pulmonary tuberculosis: A. Type of lung carcinoma that strongly correlates with smoking: A. Sun exposure ____ 93. Adenocarcinoma C. edema results primarily from: A. Bloodstream B. Papillary carcinoma of the thyroid B. Fat accumulation formation ____ 97. Berry aneurysm C. Sarcomas usually metastasize via: A. Hyponatremia D. Prostatic adenocarcinoma: Human chorionic gonadotropin C.____ 86. Dura C. X rays ____ 89. The most common pathology in the gallbladder is: A. Neuron 56 . where in the brain is Negri body located? A. Aggressive. Which contributes to the pathogensis of malignant melanoma. The mechanism of adult respiratory distress syndrome is: A. In nephrotic syndrome. Renal insufficiency with fluid accumulation C. Patients with diagnosed cavitary PTB ____ 88. Acute myelogenous leukemia ____ 90. Most common cancer among males and females D. Local extension invasion C. basal cell carcinoma and squamous cell carcinoma of the skin? A. Hemangiona Microaneurysm ____ 98. Most common cause of subarachnoid bleed? A. Which type of radiation would be most harmful? A. Which of the following relationships is a MISMATCHED PAIR? A. Gelatinous consistency C. Inflammatory reaction to microorganisms B. Exposure to ultraviolet light is most important in development of: A. Previous irradiation C. Patients with miliary tuberculosis C. Squamous cell carcinoma of the lung D. Basal cell carcinoma of the skin C. Hypocalcemia C. In the Philippines. Hypoproteinemia B. Small cell cancer of lung: Neuron – specific enolase (NSE) ____ 95. highly metastasizing B. Patients with undiagnosed cavitary pulmonary tuberculosis (PTB) D. Pure testicular seminoma: Placental alkaline phosphatase D. Urinary protein loss due to glomerular injury ____ 94. Hypergammaglobulinemia ____ 96. Chronic inflammation C. Predominantly hematogenous spread ____ 92. Inappropriate sodium and water absorption A. Bronchoaleveolar carcinoma D. Mucinous tumor B. Adenoma B. Which of the following best describes invasive ductal carcinoma of the breast? A. Diffuse damage to alveolar capillary wall D. Alpha particles B. Increased solute resulting in water retention B. A-V malformation B. In severe hepatic disease. Gamma rays D. In rabies infection. Blood vessel B. Prolonged cosmetic use B. Deficiency in pulmonary surfactant C. Children with undiagnosed primary infection B. Stone D. the edema is likely due to: A. Perineural ____ 99. Glial cells D. D. Squamous cell carcinoma ____ 91. Beta particles C. Necrosis of parenchyma 905 A 10 Most important environmental factor for pancreatic cancer A. Senile atrophy answer # question page B 1 The central factor in the pathogenesis of irrevsible cell injury would be A. C 18 A 4 The ultimate mechanism of sudden cardiac death is: A. cell death D. promotes ATP production B. histologic reaction of numerous lipophages D. B. gas gangrene B. complication of mitral insufficiency 564 A 8 In peripartum cardiomyopathy.B. Physiologic atrophy of the brain in an 82 year old male is mainly due to: A. activation of pancreatic amylases into the peritoneal cavity B. free radical formation 9 C 2 Which would be the consequence(s) of the occurance of mitochondrial permeability transition ? A.____ 100. A. large and flabby heart B. aortic valve stenosis 564 D 5 Myxomatous degeneration of the mitral valve is associated with: A. Disuse atrophy C. A. Fat necrosis D. lethal arrythmia B. prolapse of the mitral valve into the left ventricle B. ischemic cardiomyopathy C. Quinsy sore throat 367 57 . increase in the number of mitochondria C. Denervation atrophy B. fussion of the mitral commissures D. Neutrophilic infiltration B. asymmetric septal hypertrophy 580 C 9 The most characteristic histologic lesion of acute pancreatitis A. the systolic dysfunction of the patient is associated or explained by: A. Loss of endocrine stimulation D. Hyperlipoprotenemia C. Alcoholism D. impaired ventricular filling during diastole D. Smoking B. Hemorrhage C. membrane damage C. ischemic areas in the left atrial septum C.C 12 B 3 The presence of chalky. Viral infection 910 C 11 Ritter disease is also known as A. Staphylococcal scalded skin syndrome D. massive ventricular hypertrophy without dilatation C. mitochondrial dysfunction B. Hidradenitis suppurativa C. combination of fatty acids released with calcium C. white areas in the adipose tissue in the abdominal cavity would be interpreted to be associated with: A. cytoskeletal abnormalities D. dilated cardiomyopathy D. lung fibrosis C. microcytic anemia is deficient in A. Campylobacter jejuni C. cancer of nasal cavity 422 A 16 Indoor air pollutant A. Aspergillus C. Wuchereria bancrofti C. B D. marasmus C. carbon monoxide B. A1 B. A2 C. zinc B. Any of the above 389 383 373 A 15 Consequence of lead exposure A. None of the above 805 58 . Salmonella typhi B. Hirschprung disease B.B 12 Elephantiasis is associated with A. Meckel diverticulum C. asthma D. Pneumocystis carinii D. copper C. Celiac sprue D. iron 452 C 18 A child came in at the out-patient department because of generalized edema accompanied by easily pluckable hair. selenium D. None of the above 835 B 21 Grossly appears as a blind pouch on the antimesenteric side of the small bowel A. Any of the above 398 D 13 Fungi within small cysts in the Virchow-Robin perivascular space is characteristic of A. Most likely she is suffering from A. sulfur dioxide C. kwashiorkor D. bulimia 438 C 19 Traveler's diarrhea is associated with what organism A. listlessness. acid aerosol 418 D 17 A patient with hypochromic. Mucor B. Cryptococcus neoformans 380 D 14 Differential diagnosis of a child who presents with fever A. anorexia B. malaria B. Escherichia coli D. and loss of appetite. poliovirus infection D. dengue C. Onchocerca volvulus B. ozone D. Shigella 808 A 20 Astler-Coller classification when the malignant glands in the colon are limited to the mucosa A. memory loss B. Ancylostoma duodenale D. Lactotroph adenoma B. alcholism C. The most likely diagnosis is 1135 59 . enlarged thyroid gland. Hyaline arteriolosclerosis B. as squamous cell carcinoma of the esophagus C. Somatotroph adenoma D. This is usually associated with A. Acute pyelonephritis D. as acute gastritis D. presence of relatives with gastric carcinoma D. Any of the above 1058 B 27 A 30 year old female complained of loss of consciousness. Pancreatic insufficiency D. polyp C. luteal cyst D. Liver Cirrhosis C. Any of the above 799 779 783 789 1076 1069 1066 1066 B 26 Differential diagnosis of a patient with sessile masses projecting into the endometrial cavity A. Lab exam showed increased T4 and T3. PE revealed tender. dysgerminoma C. follicular cyst C. A. fever and malaise. longitudinal tears in the esophagogastric junction B. Any of the above D 23 Alcoholic patients may manifest A. Diffuse glomerulosclerosis 923 A 31 Most frequent type of hyperfunctioning pituitary adenoma A. decreased TSH. thecoma-fibroma B. lichen sclerosus D. History revealed that the episodes were precipitated by fasting. Gonadotroph adenoma 1125 C 32 A 23 year old female complained of sore throat.C 22 Information to elicit in the clinical history of a patient with gastric carcinoma A. granulosa-theca tumor D 25 Which is an ovarian cyst? A. yolk sac tumor D. Nodular glomerulosclerosis C. Any of the above C 24 Schiller-Duval body is associated A. Somatostatinoma 926 D 29 A newborn infant was born with a hemolytic disease due to RH incompatibility. Anasarca B. Glucagonoma D. papillary hidradenoma B. Corticotroph adenoma C. Gastrinoma B. Kernicterus 474 B 30 The most characteristic renal finding in diabetes mellitus A. Insulinoma C. Laboratory exam at the time of attack showed glucose level at less than 50mg/dl. fondness for barbecued foods B. serous cyst B. What is the most serious threat in this disorder. Radial growth phase B. De Quervain thyroiditis D. Neuroblastoma B. Serous B. Anemia B. Cystitis glandularis D. It has a benign nature but has tendency to bleed profusely during surgery. Engulfment C. Rosenthal fibers 1373 A 34 Microscopic features of this lesion in the urinary bladder revealed infiltration with large foamy macrophages with multinucleated giant cells interspersed with lymphocytes. PUVA 1185 60 . Blastema cells D. Inverted papilloma 877 A 37 Step/s in phagocytosis enhanced by opsonins: A. These are also called Adenolymphomas B. Reidel thyroiditis B 33 The most characteristic microscopic finding of Retinoblastoma is: A. All 1179 A 41 The most commonly accepted cause of squamous cell carcinoma of the skin is: A. Squamous cell carcinoma C. A & B only 62 D 38 Abnormal acquired leukocyte chemotaxis is/are seen in: A. Suppurative C.A. Homer Wright rosettes B. Tuberculosis C. Exposure to arsenic C. Cutaneous scars D. Killing or degradation D. Angiofibroma D. Michaelis Gutmann bodies are present A. Recognition & attachment B. Sepsis C. Acute suppurative cystitis 1002 B 35 Which of the following is TRUE of Pleomorphic adenoma A. A. Malakoplakia B. Chronic granulomatous 83 B 40 The biologic behaviour of malignant melanoma is determined by: A. There are the most common tumors of the salivary gland C. Flexner Wintersteiner rosette C. B & C only 65 C 39 Distinctive pattern of inflammation seen in tuberculosis A. Graves disease B. These are highly malignant tumors D. Hashimoto thyroiditis C. Diabetes mellitus D. Degree of pigmentation D. Occurs much more often in males 770 C 36 This is a highly vascular tumor of the nasopharyngeal area that occur almost exclusively in adolescent males. Nature & extent of vertical growth phase C. Fibrinous D. Exposure to ultraviolet light B. follicular lymphoma B 50 Hypochromic Microcytic RBC is seen in : A.Ab-mediated cellular dysfunction D.immune complex mediated 199-200 C 45 True of clonal anergy : A. Vitamin B12 deficiency B. suppressor T-cells down regulating the other T-cells 213-214 C 46 A 53 y/o G10P10(10-0-0-10) female underwent transplantation of the right kidney. the transplanted kidney become cyanotic. clonal deletion of self-reactive T-cells during maturation C. Anti-Scl 70 C. Type II . acute lymphoblastic lymphoma D. Iron deficiency anemia C. Fas-FasL engagement leading to apoptosis of activated T-cells B. CCR4 B. follicular hyperplasia C. smudge cells C.delayed hypersensitivity B. Anti-centromere antibody B.B 42 The detection of this antibody is specific for systemic sclerosis : A. direct cell mediated reaction B. Ab-mediated reaction D. What is the mechanism of rejection? A. CD10 and surface IgM. Reed-sternberg cells C 48 Left enlarged axillary lymph node from a 45 y/o female with left breast carcinoma would likely show : A. Give the probable diagnosis : A. eosinophil B. there is no graft rejection 207-208 D 47 The true neoplastic element in Hodgkin disease : A. Tumor cells express CD19. paracortical hyperplasia B.complement dependent C. Anti-Sm 227 C 43 The receptor used by the HIV strain that is effective in transmitting the infection : A. small lymphocytic lymphoma C. Anti-DsDNA D. Fifteen minutes after transplantation. indirect cell mediated reaction C. Type IV . Hereditary spherocytosis 670 649-650 662 H-619 61 . plasma cells D. Burkitt's lymphoma B. Type II . mottled and flaccid. Type III . Sickle cell disease D. Ag is presented by cells not bearing the proper ligand resulting to a negative signal D. sinus histiocytosis D. CXCR4 C. CXCR5 240 B 44 Blood transfusion reaction is an example of which mechanism of hypersensitivity reaction ? A. CCR5 D. The donor kidney was taken from her husband. histologically normal A 49 Incision biopsy of a submandibular mass of a 6 y/o African boy revealed diffuse infiltrates of intermediate-sized lymphoid cells with "starry sky" pattern. decrease cell death 90 D 56 Grossly. large cell caricnoma 745 62 . infection persists B. chest pain C. tumor frequently metastasize to the CSF C. adenocarcinoma C. tubular adenoma 1117 B 58 Which is TRUE in a patient diagnosed with invasive lobular carcinoma? A. True of this condition : A. Colloid carcinoma 1112 B 57 Which is a common benign lesion in male breast? A. tissue remodelling 103 C 54 Vitamin C deficiency retards wound healing because: A. tumor cells are confined to the lobule D. wound contraction is inhibited C. large lobes of lightly staining mucin with floating neoplastic cells are seen. decrease the cell population B. squamous cell carcinoma D. weight loss B. cough D. collagen synthesis is inhibited D. Tubular carcinoma C. a breast tumor is soft with an appearance of pale gray-blue gelatin. lagre quantities of Hb A D. decrease cell proliferation D. <5% hemolysis C. marked anemia and hepatosplenomegaly shows large quantities of Hb Bart's. fibroadenoma B. what is the most probable histologic type? A. complete absence of beta chains C.D 51 Sucrose hemolysis test result in a patient with paroxysmal nocturnal hemoglobinuria : A. prominent lymphoplasmacytic infiltration 1111 C 59 What is the most common complaints of lung cancer patients? A. Schirous carcinoma B. exuberant granulation occurs 110 A 55 Increased apoptotic activity will: A. 5-10% hemolysis D. >10% hemolysis H-635 A 52 A stillborn infant with severe edema. What is your diagnosis? A. vasculogenesis C. oat cell carcinoma B. Microscopically. Medullary carcinoma D. the tumor is always unilateral B. gynecomastia C. fibrocystic change D. granulation tissue formation D. no hemolysis B. complete absence of alpha chains B. increase cell differentiation C. angiogenesis B. dyspnea 745 B 60 A female patient with no previous smoking history is diagnosed to have bronchogenic carcinoma. small quantities of Hb F H-647 B 53 Angioblasts are involved in: A. ras gene chromosome 12p 296 63 . primary atypical pneumonia D. p53 B. & miosis. painful cramps B. Sphingomyelinase C. middle lobe C. BRCA1 286 C 68 Tumor composed of more than one neoplastic cells type from more than one germ layer A. & adjacent alveolar spaces are characteristics seen in: A. apex D. Ib C. Ia B. bronchopneumonia 721 B 64 B 65 Migration of tumor cells in invasion of extracellular matrix is facilitated by A. alpha 1-iduronidase 155 D 70 Characteristic clinical manifestation of Von Gierke's disease A. bronchioles. lower lobe B. Glucocerebrosidase D. whole lung 747 C 62 A malignant lung tumor is 3 cm in size without pleural involvement & has ipsilateral hilar node involvement. HPV 11 D. metalloproteinases B. massive cardiomegaly 160 The associated molecular change in the evolution of colorectal tumor from late adenoma to carcinoma is loss of A. fibroadenoma C.C 61 A patient presented with enophthalmos. leiomyoma 261 A 69 Enzyme deficiency in Tay Sacch's disease A. He was previously diagnosed with lung cancer. IIb 745 D 63 Patchy consolidation of the lungs & the presence of suppurative exudate filling up the bronchi. HPV 16 B. lobar pneumonia C. IIa D. tumor suppressor gene chromosome 18q D. Where is the most probable site of the tumor? A. Hexosaminidase A B. thymosin B15 c. tuberculosis B. cadherin D. APC gene chromosome 5q B. WT1 C APC D. what is the stage of the tumor? A. p53 chromosome 17p C. HPV 4 311 D 67 Tumor suppressor gene involved in breast carcinoma A. pleomorphic adenoma B. Mature cystic teratoma D. HPV 6 C. beta catenin 302 A 66 High risk HPV involve in cervical carcinoma A. ptosis. congestive heart failure 526 C 84 The most characteristic histologic lesion in the glomeruli of patients with Rapidly progressive glomerulonephritis is: A. 114-116 951-952 64 . hypovolemic shock C. bacterial dissemination. binding D.Epithelioid Histiocyte B. in contrast to red infarct. 130 D 83 A 57 year old seaman was admitted because of dysphagia. synthesis 152 B 75 What is the most characteristic cell found in Rhabdomyoma? A. skeletal abnormality B. dyspnea.The patient later developed a septic pulmonary infarc-. rashes. sepsis D. café au lait spots D. hypoglycemia C 71 Most threatening feature of Marfan's syndrome A. paradoxical thromboembolisation D. transport B. Myotonic Dystrophy C. ocular findings C. and chronic cough of 6 months duration.He was treated for a penile chancre followed by fever. solid B. Becker Muscular Dystrophy D. increase osmotic pressure C.C. vascular permeability D. Most patients with this lesion die of: A. sternal pain. with double blood supply C. cardiovascular lesion D. hyperextensible skin 149 C 72 Class III mutation in Familial hypercholesterolemia is a defect in A. cardiogenic shock B. and lymphadenpathy 30 years ago. previously congested. is usually seen in organs which are: A. B 78 C 79 A 24 years old male drug abuser developed Subacute bacterial endocarditis involving the mitral valve.Spider Cell C. recycling C. pulmonary thromboembolisation C. Duchenne Muscular Dystrophy B. basement membrane thickening C. through a process known as: A. focal hyalinosis B. systemic thromboembolisation B. A and C only A 77 A white infarct.Mallory Cell D. increase oncotic pressure B.Langhan's Giant Cell 591 D 76 Associated with little or no production of the 427kD protein called Dystrophin: A. infected D. crescent formation 12811283 132 Patients with left-sided heart failure may develop pulmonary edema because of which of the following mechanisms? A. sodium retention. Lewy bodies D. angiosarcoma C. with pubic and axillary hair and husky voice. Seminoma B. hypercellularity. B 89 D 90 C 91 Characteristic of CNS neoplasia A. defective transport D. space occupying lesion in the brain D. Pertinent physical examination findings revealed gynecomastia. majority arise form neurons C. metastatic cancers are uncommon 1343 D 92 Hydrocephalus may result from A. Acute tubulointerstitial nephritis. metastasize early B. decreased brain parenchyma C. Erythroplasia of Queyrat C.200 ml/24 hours. tuberculous meningitis B.D. C 86 A 28 year old primigravida had hypotensive shock because of postpartum hemorrhage. Her urine output ranged from 80 . site is more important than grade D. all of these 1298-9 C 96 Liver cancer seen in young adults without predisposing factors A. Embryonal cell carcinoma 1024 Found in the hippocampus and purkinje cells of the cerebellum of patients with Alziemer's Disease A. arranged in disorderly fashion with increasd mitosis. Bowenoid papulosis D. Squamous cell carcinoma. Leydig cell tumor C. brown atrophy B. Sertoli cell tumor D. cholangiocarcinoma B. Hirano bodies 1331 65 . Russell bodies D. Psammoma bodies C. The most probable testicular lesion is: A. and confined within the basement membrane of the penile skin is consistent with: A. hepatic adenoma 890-1 C 98 Mechanism of triglyceride accumulation in malnutrition A. Cowdry bodies B. Acute tubular necrosis D. decreased enzyme 39 C 99 Protein accumulation in cell A. exogenous accumulation C. fibrolamellar HCC D. Hemolytic-uremic syndrome B. Bowen's disease B. Negri bodies C. 969-971 A 88 Histologic findings of markedly enlarged hyperchromatic nuclei. steatosis 41 1013 An 8 year old male child was brought to the hospital for testicular enlargement. excessive production B. Obstructive uropathy C. The basic kidney lesion to explain this decreased urine output among these patients is: A. Collins 6th edition H= Clinical Diagnosis and Management by Laboratory Methods by Henry nineteenth edition 1. 693 b. coagulation necrosis c.C 100 Most common initial outcome of acute inflammation in Peptic ulcer: A. liquefaction d. Marked thrombocytopenia and a DIC-like phenomenon are responsible for the unusually severe bleeding tendency in which of the following: a. Progression to chronic inflammation D. Acute monocytic leukemia c. A 60 year-old woman presented with postmenopausal bleeding. 646 66 . Hodgkin’s lymphoma ans. C p. median 6. Acute promyelocytic leukemia 8. A p. phospholipids c. Obstruction of the blood supply to the brain would lead to which type of necrosis: a. 25 b.Kumar. Total abdominal hysterectomy and bilateral salpingo-oophorectomy was done. plasma cell 5. The histopathologic examination of the left ovary would most likely show which of the following: a. B p. Pelvic ultrasound showed a solid masss in the left overy. Granulose cell tumor ans. Mucinous cystadenoma d. What is the cell of origin of the epitheloid cell and the Langhan’s giant cell seen in tuberculous lesion: a. Healing through fibrosis C. Mature teratoma c. a p. lateral d. posterior c. monocyte c. caseation necrosis ans. C p. Neutrophils ans. Resolution 79 REFERENCE Robbins Pathologic Basis of Diseases by Cotran. cholesterol/cholesterol esters d. Chalky-white appearance of necrotic tissues is seen in which of the following: a. enzymatic fat necrosis d. B p. liquefaction necrosis 3. Acute lymphocytic leukemia ans. Fibroma 7. c p. 1102 b. Lacunar cells are seen in which of the following: a. D and C showed endometrial hyperplasia. Adenocarcinoma of the prostate tends to involve initially which of the following lobes of the prostate gland: a. 138 b. 216 b. lymphocyte d. Anterior ans. coagulation c. Caseation ans. enzymatic fat necrosis 4. Which of the following accumulates in the cytoplasm of liver cells in steatosis: a. Sphingolipids ans. d p. Abscess formation B. 1054 b. 1092 b. triglycerides 2. Acute myelocytic leukemia d. bleeding 10. hematogenous 67 . Plasma cell dyscrasia d. Deficient pulmonary surfactant c. Non-Hodgkin’s lymphoma c. The most common cause of death in Multiple Myeloma is : a. involvement of the lower third of the vagina is what stage. Acini 14. Oxygen metabolites 16. 1077 b. widespread metastases c. IV 15. Langerhans cell histiocytosis 9. 279 b. D p. Diffuse damage to alveolar capillary walls ans. D p. a. Kimmelstiel-Wilson’s lesions in diabetes mellitus are seen in which of the following: a. the resultant scar is much smaller than the original wound because of which of the following: a. The cell or tissue of origin of most breast carcinomas is the: a. D p. I ans. B p. 1215 b. Which of the following is the most common pathway for the initial route of metastasis of carcinomas: a. Conn’s syndrome 13. kidney 12. Duct c. C p. Herpesvirus ans. D p. Lobule d. In healing by secondary intention. Formation of hyaline membranes 18. Histamine ans. Eyes ans. Which of the following is the most likely mediator of pain: a. infection d. Cytokine d. Waterhouse-Friederischen syndrome c. Less fibrin formation ans. More granulation tissue d. Wound retraction 17. c p. N. Diffuse infiltration of the adrenal gland is a pathologic lesion most commonly associated with which of the following: a. 715 b. 664 b. 68 b. C p. II c. C p. The final common pathway of ARDS is which of the following: a. pancreas c. liver d. seeding c. renal failure ans. Addison’s disease d. Stroma ans. Plasma cells are predominant cellular infiltrates in lesions associated with which of the following: : a. Fibrin exudation d. direct extension ans. B p. Cushing’s syndrome ans. Removal of exudates c. gonorrheae c. III d.b. pallidum d. . Prostaglandin c. Hepatitis B virus 11. 991 b. 113 b. 388 b. In cervical carcinoma. 1130 b.T. A p. B p. Blood vessels 29.d. Movement of leukocytes towards the site of injury is: a. D p. nitric oxide 23. Which of the following is the substance that is responsible for massive peripheral vasodilatation in sepsis: a. systemic circulation 21. D p. in the form of crescents. Emboli of major veins of the legs will eventually plug small vessels of which of the following: a. Viral hepatitis ans. C p. A p. Nephritic syndrome 28. Primarily unconjugated hyperbilirubinemia is seen in which of the following: a. Lupus nephritis 27. currant jelly clots c. coronary circulation c. 130 b. A p. 56 b. Cells which act as vehicles in the transport of HIV to the other parts of the body: a. Small defect 25. Transmigration 24. A p. Chemotaxis c. C p. pulmonary circulation ans. 136 b. 976 b. in the bowman’s space is characteristic of which of the following: a. 72 b. Hematuria. Tubules c. 888 68 . 974 b. both d. lines of Zahn ans. lymphatics 19. 113 b. Neutrophils c. Cytokines ans. cerebral circulation d. Idiopathic rapidly progressive glomerulonephritis ans. 981 b. A p. neither 20. Stasis ans. endothelial injury d. Wound contration ans. B p. Plasma cells 26. Diffuse effacement of foot processes is characteristic of: a. Rapidly progressive glomerulonephritis d. Death of limited cells d. Lymphocytes d. Secondary union is characterized by: a. 133 b. Which of the following is the most important factor in the development of thrombosis: a. Monocytes ans. oliguria and hypertension indicate involvement of the: a. Opsonization ans. Minimal tissue loss c. Focal segmental glomerulosclerosis ans. 248 b. hypercoagulability of blood 22. turbulence c. fibrin degradation products d. Membranous glomerulonephritis c. Which of the following are characteristics of antemortem clots: a. Lipoid nephrosis c. Diapedesis d. oxygen-derived free radicals c. Glomeruli ans. Accumulation of cells. Interstitium d. IgA nephropathy d. Barrett esophagus ans. Reflux esophagitis d. B p. Most deaths that occur during acute rheumatic fever are caused by which of the following: a. Endocarditis 38. D p. The classic diagnostic features of costovertebral pain. A p. Anti. 549 b. Alzheimer’s disease c. Dubin-Hohnson syndrome c. Neuronophagia d. HEV 34. 891-892 b. H. b p. b p. Brown atrophy ans. Gilbert’s syndrome 30. Atrial septal defect 37. 1018 b.HBe ans. Variceal dilatation of the anal and perianal plexuses is called: a. Huntington’s disease 69 . b b. Pericarditis c. Persistent infection and chronic hepatitis are hallmarks of which of the following: a. Streptococcal sepsis ans. Arteriovenous malformation 36. 854 b. 1351 b. Wilm’s tumor c. Patent duotus arteriosus ans. Ventricular septal defect d. EXCEPT: a. Urethelial CA of the renalpelvis d. Hemangioma d. Corpora amylacea 32. B p. A p. pylori is associated with which of the following conditions: a. Rosenthal fibers ans. HBV ans. The following are effects of aging in the heart. HCV c. Angiodysplasia ans. Anti HBsAg 33. Tetralogy of Fallot c. Hemorrhoids c. palpable mass and hematuria suggest the presence of which of the following: a. Which of the following markers confer protection and forms the basis for current vaccination strategies: a. Urinary bladder CA 31. Biliary tract obstruction d. Lambl’s excrecences p. “sigmoid” septum d. Peptic ulcer c. Parkinsonism ans. Which of the following is associated with early cyanosis: a. IgG anti-HBe d. B p. “boot”-shaped heart c. 575 b. Coinfection with HDV and HBV d. Which of the following primarily affects the cerebral cortex: a. Myocarditis d. 519 39. Gliosis c. 817 b.b. IgM anti HBe c. 894 b. Curling’s ulcer 35. 1329 b. c p. Renal cell carcinoma ans. The most important histopathologic indication of CNS injury is: a. 923 b. Middle third c. b b. a b. post pneumonic complication d. The most common cause of hydrothorax is: a. 1053 b. Urine amylase d. Serum lipase c. Upper third ans. Nephritic syndrome d.d. Gastrinoma c. Serum amylase ans. Bronchitis ans. c p. Which of the following is the classic physiologic feature of diffuse interstitial lung disease: a. Glucagenoma 42. 786 b. lung volume or compliance 49. Most cases of esophageal carcinomas are located in the: a. pancreatic islet tumor. The most common cause of death in patients with advanced cervical carcinoma is: a. Infection c. cecum d. 729 70 . ileum c. b p. Emphysema c. Widespread metastasis d. 1072 b. peptic ulcer disease unresponsive to treatment and severe diarrhea characterize which of the following: a. The most frequent mechanism in the development of lung abscess is: a. Teratoma c. Cirrhosis p. Gastric hypersecretion. b p. b p. 692 b. direct trauma 48. Congestive heart failure c. Uremia ans. Chronic necrotizing infection of the bronchi and bronchioles associated with abnormal dilation of the airways is: a. septic embolism ans. c p. 765 b. Decreased pulmonary resistance c. Dysgerminoma 46. 699 b. Serum calcium 44. Limited expiratory flow rates d. Bronchiectasis d. Multiple endocrine neoplasia d. Insulinoma ans. Endodermal sinus tumor d. 901 b. a p. Bleeding 45. This is elevated up to 2 weeks in acute pancreatitis: a. Struma ovarii is a variant of which of the following: a. Renal failure ans. Hirschsprung’s disease usually involves which segment of the intestine: a. rectum 43. jejunum ans. Reduction in oxygen diffusing capacity. Gastroesophageal junction 41. Friedrich ataxia 40. b p. Increased lung volume ans. c p. Lower third d. aspiration of infected material c. Lung abscess 47. Granulosa cell tumor ans. peripheral nervous system c. Draining sinuses d. Scirrhous carcinomas of the breast are usually: a. d p. 1320 b. pons c. ductal carcinomas p. both d. neither p. a p. 1230 b. a b. In rabies. The most common site involved in hypertensive intracranial hemorrhage is which of the following : a. 417 51. c p. From a focus in the pelvis c. infarction of brain tissue ans. b p. Coup and contrecoup injuries are characterized by which of the following: a. medullary carcinomas ans.50. The involucrum of osteomyelitis conssts of which of the following: a. Thalamus ans. Which of the following is associated with osteoporosis: a. Which of the following is correct regarding how Tuberculous osteomyelitis arise: a. pons 56. putamen d. 1320 b. from a hematogenous dissemination 53. contusion of brain 55. from a focus in the pelvis c. The anterior horn motor neurons of the spinal cord are primarily affected in which of the following conditions: a. cytomegalovirus infection ans. Negri bodies are found in which of the following: a. 1220-1221 54. increased parathyroid hormone c. 1305 b. herpes simplex encephalitis 59. Osteosclerosis c. Pectus excavatum p. reduced physical activity ans. Which of the following is seen in rickets: a. d p. 1231 b. Reactive bone around a sequestrum 52. local extension d. Hippocampus ans. temporal lobes c. b b. spontaneously ans. Bowing of the bones ans. d p. cerebellum 57. Normal growth zones in metaphysic d. poliomyelitis c. toxoplasmosis d. 1311 b. hypothalamus d. direct implantation ans. hemorrhage of brain tissue c. hematogenous 58. c b. 1103 71 . from a focus in the spinal cord d. b p. What is the most common route of entry of infectious organisms in the central nervous system: a. laceration of brain tissue d. 1314 b. Necrotic bone ans. Bacterium ans. c p. What kind of organism is Pneumocystis carinii: a.c. all of the above 61. schistosomiasis d. fungus p. tuberculosis c. ascending colon ans. rectum 63. more numerous d. fibrocystic disease 62. Which of the following is/are characteristic mammographic calcifications in breast carcinomas: a. The characteristic flask-shaped ulcers in amebiasis are most frequently seen in which of the following: a. Smaller ans. Which of the following is the most common cause of lump in the breast: a. 228 64. d p. fibroadenoma d. protozoa d. cancer c. malaria 72 . more tightly clustered c. descending colon d. 1106 b. Syphilis ans. Pipestem fibrosis of the liver is seen in which of the following: a. 333 b. d p. 1091 b. virus c. a p. 372 b. lobular carcinomas 60. c b. no disease ans. mucinous carcinomas d. transverse colon c.
Copyright © 2024 DOKUMEN.SITE Inc.